Categories
Chicago Exam Questions

Chicago. Final exam for Economics 301, Price Theory. Telser, 1965

 

Chicago Price Theory boils down ultimately to a series of True-False-Uncertain examination questions. One of Lester Telser‘s contributions to the stock of questions comes to us from the Zvi Griliches’ papers at the Harvard Archives. A steady diet of this stuff would make for a dull economist in my opinion, but it was force fed to generations of Chicago economists, and many somehow survived to have productive (in a good sense) professional careers. Hence another important historical artifact that has earned digitization by Economics in the Rear-View Mirror.

______________________________

UNIVERSITY OF CHICAGO
DEPARTMENT OF ECONOMICS
Final Exam

Economics 301
Autumn, 1965

Mr. L. Telser
Time: 2.5 hours

Answer the following questions, true, false or uncertain and briefly defend your answer.

  1. An increase in the demand for the product of a monopoly results in a rise in the price of the product.
  2. If the supply schedule of an industry is perfectly elastic then the production function for the industry is characterized by constant returns to scale.
  3. A monopoly can never have a larger output and lower price than a competitive industry assuming that cost conditions would be the same for both.
  4. No one would resort to the cultivation of inferior lands if he did not run into diminishing marginal returns on fertile land. Since inferior lands are in cultivation, diminishing returns must be present.
  5. If there are empty seats on a train then marginal cost pricing requires that new passengers should ride free.

The following two questions require essay answers.

  1. The margin in stock trading is the fraction of the price of the stock which the trader must supply and the balance is lent to the trader by the broker. Hence the margin represents the trader’s equity and is analogous to a down payment. Assume there is no government regulation of the margin and that brokers are free to set any margin they please and to charge any interest rate they please on the loan they extend to traders. Assume there is perfect competition in the brokerage industry.
    1. Would you expect margins to be higher during periods of “active” speculation?
    2. Would you expect higher margins when stock prices are rising then when they are falling?
    3. Would a rise in the interest compensate for or be equivalent to a rise in the margin?
  2. In the theory of the household demand for perishable goods, for a given money income there is a fall in real income if the price of some good rises. The pure substitution effect is the effect on quantity demanded of a price change for constant real income. In the case of durables households own stocks of durables. Hence a rise in the price of durables causes the value of the stock of consumer owned durables to appreciate. Hence a price rise of perishable reduces the demand for perishables if money income is given and the income elasticity is positive while a price rise of durables increases the demand because it implies a rise in consumer wealth if the wealth elasticity of demand is positive.
    1. What are appropriate budget constraints for the demand for durables?
    2. Is the last statement beginning with “HENCE …” correct?
    3. Would it make a difference if the durable good had a fixed life or if it lasted forever?
    4. What are the counterparts of constant real income in the demand for durables?

Source: Harvard University Archives. Papers of Zvi Griliches. Box 130, Folder “Syllabi and exams, 1961-1969”.

 

Categories
Chicago Exam Questions Suggested Reading Syllabus

Chicago. Course outline, readings, examination for introduction to econometrics. Marschak, 1949

The following course material was transcribed from copies found in Franco Modigliani’s papers at the Economists’ Papers Archive in the David M. Rubenstein Rare Book & Manuscript Library of Duke University. These items are also available in a scanned .pdf file at the Cowles Foundation website at Yale. Modigliani’s original mimeographed copy is for the most part much more legible than the on-line scanned copy at the Cowles Foundation. This is particularly true for the “terminal examination” questions. Over forty pages of typescript for the lectures are also found in the original Cowles Commission Discussion paper.

More on Jacob Marschak can be found in Robert W. Dimand’s “Keynesian Economics at the Cowles Commission” (Review of Keynesian Studies, vol. 2, 2020, pp. 22-25).

________________________

J. Marschak. INTRODUCTION TO ECONOMETRICS
Economics 314
Spring 1949.

314. Introduction to Econometrics: Statistical testing of economic theories. Numerical estimation of demand and cost functions and other functions occurring in the theory of the firm and household, the theory of markets and the theory of national income. Estimation of economic models. Statistical prediction under conditions of changing economic structure and policy. Prerequisites: Econ 310, 311, 312 or equiv. Win [sic] TuTh 3-4:30; Marschak.

Source: University of Chicago.The College and the Divisions, Sessions of 1948-1949. In Announcements Vol. XLVIII (May 25, 1948) No. 4, p. 250.

________________________

INTRODUCTION TO ECONOMETRICS
20 Lectures given at the University of Chicago in Spring, 1949*

Cowles Commission Discussion Papers, Economics: 266

[*To be used jointly with 24 Lectures (same title) given at the University of Buffalo in Spring, 1948.]

Part I. Non-stochastic economics [11 lectures]

  1. Best policy. Goal variable; non-controlled, controlled, strategic variables.
  2. Exogenous variables and structural parameters. Types of prediction.
  3. Determining the structure from theory and data.
  4. An example.
  5. Econometric “pitfalls” due to disregarded variables or relations. Non-idenifiable structures.
  6. [continued]
  7. The identification, continued
  8. Why does the identification problem arise in non-experimental sciences?
  9. Discussion of earlier problems.
  10. Discussion of earlier problems. [continued]
  11. When need we know the structure?

Part II. Stochastic economics: Population properties [8 lectures]

  1. Joint distributions, non-parametric.
  2. Mid-term examination.
  3. Parameters of joint distributions.
  4. Least-squares property of coefficients of linear regression. Properties of normal distributions.
  5. Exogenous and endogenous variables in stochastic economics.
  6. Identification and determination of structure by the method of reduced form: examples.
  7. More examples.
  8. Motion of an economic variable. Dynamic models. The assumption of independent successive disturbances and its implication.

Part III. Stochastic economics: Sample properties [1 lecture]

  1. Useful properties of certain least squares and maximum likelihood estimators. Obtaining maximum likelihood estimates of structure from those of reduced form.
Recommended reading.
Attached Materials**

J. Marschak, “Economic Structure, Path, Policy and Prediction”

__________, “Statistical Inference from Non-Experimental Observations—an Economic Example”

G. Hildreth, “Problems in the Estimation of Agricultural Production Functions”

[**As far as available.]

*     *     *

READING MATERIAL TO BE USED IN COURSE ON INTRODUCTION TO ECONOMETRICS,
SPRING QUARTER, 1949

  1. Allen, R. G. D., Mathematical Analysis for Economists.
  2. American Economic Association, “Survey of Contemporary Economics” (Blakiston Co., 1949).
  3. Haavelmo, T., “The Probability Approach to Econometrics” (Supplement to Econometrica, 194) .
  4. Haavelmo, T., “Quantitative Research in Agricultural Economics,” Journal of Farm Economics, Vol. 29, No. 4, November, 1947.
  5. Girshick, M. A., and T. Haavelmo, “Statistical Analysis of the Demand for Food,” Econometrica, Vol. 15, No. 2. April, 1947.
  6. Klein, Lawrence R., “The Use of Econometric Models,” Econometrica, April, 1947.
  7. Haavelmo, T., “Methods of Measuring the Marginal Propensity to Consume,” Journal of the American Statistical Association, March, 1947.
  8. Klein, Lawrence R., “A Post-Mortem on Transition Predictions,” Journal of Political Economy, August, 1946.
  9. Marschak, J., L. Hurwicz, Abstracts of papers: Econometrica, April, 1946, pp. 165-170.
  10. Koopmans, T., “Statistical Estimation of Simultaneous Economic Relations,” Journal of the American Statistical Association, Vol. 40, December, 1945.
  11. Marschak, J. and William H. Andrews, “Random Simultaneous Equations and the Theory of Production,” Econometrica, Vol. 12, No. 3-4, July-October, 1944.
  12. Marschak, J., “Money Illusion and the Demand Analysis,” The Review of Economic Statistics, 25, February, 1943.
  13. Marschak, J., “Economic Structure, Path, Policy, and Prediction,” American Economic Review, Vol. 37, May, 1947, pp. 81-84. Lil.
  14. Marschak, J., “Statistical Inference from Non-Experimental Observations,” Econometrica, January, 1948, p. 53.
  15. Hurwicz, L., “Some Problems Arising in Estimating Economic Relationships,” Econometrica, Vol. 15, July, 1947.
  16. Tinbergen, J., “Business Cycles in the U.S.A. 1919-1932” (Statistical Testing of Business-cycle Theories. II), League of Nations, Geneva, 1939.
  17. Koopmans, T., “Measurement without. Theory,” Review of Economic Statistics, 29, August, 1947.

*     *     *

J. Marschak.
INTRODUCTION TO ECONOMETRICS
Economics 314, Spring 1949.

Terminal Examination

Note: Try to answer all 4 problems first, omitting the questions (III) in problems 1 and 4. Answer the questions (III) if time remains.

Problem 1. The quantity x and the price p of a perishable farm product (each measured from its population mean) are determined as in the following model (subscripts indicate time):

(1.1) Demand: xt = αpt + ut

(1.2) Supply: xt = βpt-1 + ut

(1.3) The disturbance ut is not autocorrelated; nor is vt.

Show

(I) How to estimate α for a long time series.

(II) What other structural parameters are present?

(III) (If time remains): How would you estimate those other parameters?

Problem 2. The model of the previous problem is modified as follows:

(2.1) Demand: xt = αpt + ut  (ut not autocorrelated)

(2.2) Price fixation: p_{t}=p^{\ast }_{t} , a level fixed every year by decree.

Show

(I) How to estimate α and σuu?

(II) Is the estimate of α the same as in the previous problem?

Problem 3. National income y, consumption c, and annual (saving) investment i are all measured in dollars of constant purchasing power, and

(3.1) c = αy + β + u ;

(3.2) E(u);

(3.3) i = y – c (an identity);

(3.4) i is exogenous.

(I) Show how to estimate α, β, σuu from a long time series of data on y, c, i.

(II) Suppose y, c, i denote the income, consumption and saving of an individual family which can control its savings but not its income. How does this modification affect the model and the estimation procedure from a time series of family data, or from a survey of a large number of families?

Problem 4. A survey of very large number T of firms belonging to the same industry but located in places with different wage-rates w1, …, wT has been made. The price p of the product is the same for all firms. Wage-rates and price are fixed independently of the firms’ action. The output Xt of each firm depends on labor used only, Nt, according to the formula

(4.1) {X_{t}=B_{t}N^{A}_{t}}C , t = 1, …, T

(the elasticity A being the same for all firms.) Hence,

(4.2) xt = bt + Ant, t = 1, …, T

where the small letters (except for t) stand for the logarithms. Further assume that each firm pushes its output to the point where, apart from a random deviation, the ratio \left( w_{t}/p\right)  \equiv R_{t} equals the labor’s marginal product,

(4.3) Rt = (dXt/dN)⋅Ct, t = 1, …, T

where Ct is a random percentage deviation. Hence

(4.4) Rt = R_{t}=AN^{A-1}_{t}\cdot B_{t}C_{t} , t = 1, …, T

(4.5) rt = a + bt + ct + (A-1)nt, t = 1, …, T,

where again small letters indicate logarithms.

Questions:

(I) How to estimate A?

(II) What other structural parameters are present?

(III) (If time remains): How to estimate those?

Source: Duke University. David M. Rubenstein Rare Book & Manuscript Library. Economists’ Papers Archive. Franco Modigliani Papers, Box T1, Folder “Jacob Marschak’s Courses, 1940-1949.”

Image Source: Carl F. Christ. History of the Cowles Commission, 1932-1952

Categories
Courses Exam Questions Harvard

Harvard. Economics Course Descriptions, Enrollments, and Exams. 1897-98

 

For the academic year 1897-1898 we are blessed with ample records for the economics courses offered (and bracketed) at Harvard. Detailed course descriptions, enrollment figures, semester-end exams are available and have been transcribed below for almost every course.

_______________________

ECONOMICS.
GENERAL STATEMENT.

Course 1 is introductory to the other courses. It is intended to give a general survey of the subject for those who take but one course in Economics, and also to prepare for the further study of the subject in advanced courses. It is usually taken with most profit by undergraduates in the second or third year of their college career. It may be taken with advantage in the second year by those who are attracted to political and social subjects. A knowledge of general history (such as is given in Course 1 in History) is a useful preparation.

The advanced courses divide themselves into two groups. The first group contains Courses 2, 3, 13, 14, 15, which are concerned chiefly with economic and social theory. Courses 2 and 15 follow the development of economic theory from its beginnings to the present time, with critical examination of the conclusions reached by economists of the past and the present. Course 13, on scope and method in economic investigation, continues the same subjects; it is taken to best advantage after either 2 or 15. Course 3 considers the wider aspects of economic and social study, and reviews the progress of sociological inquiry. Course 14 takes up the history and literature of socialistic and communistic proposals, and leads to a discussion of the foundations of existing institutions.

The second group contains the remaining courses, which are of a more descriptive and historical character. In all of them, however, attention is given to principles as well as to facts, and some acquaintance with the outlines of economic theory is called for.

Before taking any of the advanced courses, students are strongly advised to consult with the instructors. Courses 2, 3, 4, 7, 10, 12, 13, 14, 15, 16 may not be taken without the previous consent of the instructors. It is advised that Course 1 be taken in all cases as a preparation for the advanced courses; and such students only as have passed satisfactorily in Course 1 will be admitted to Courses 2, 3, 4, 12, 13, 14, 15, 16. But Courses 5, 7, and 9, may also be taken by Juniors and Seniors of good rank who are taking Course 1 at the same time; Course 6 is open to students who have taken or are taking cither History 13 or Economies 1; and Courses 10 and 11 are open to students who have passed satisfactorily either in History 1 or in Economics 1.

The Seminary in Economics is intended primarily for Graduate Students; but Seniors in Harvard College, who have had adequate training in the subject, may be admitted to it.

Source: Harvard University. Faculty of Arts and Sciences. Division of History and Political Science Comprising the Departments of History and Government and Economics, 1897-98,  pp. 30-31.

_______________________

Outlines of Economics
Economics 1

I. Outlines of Economics. —Principles of Political Economy.— Lectures on Social Questions and Monetary Legislation. Mon., Wed., Fri., at 9. Professor  [Frank William] Taussig, Asst. Professor Edward Cummings, Dr. John Cummings, assisted by Messrs. [Charles Sumner] Griffin, [Edward Henry] Warren, and ——.

Course 1 gives a general introduction to economic study, and a general view of Economics sufficient for those who have not further time to give to the subject. It begins with a consideration of the principles of production, distribution, exchange, money, and international trade, which is continued through the first half-year. In the second half-year, some of the applications of economic principles and some wider aspects of economic study are taken up. Social questions and the relations of labor and capital, the theory and practice of banking, and the recent currency legislation of the United States, will be successively treated in outline.

Course 1 will be conducted mainly by lectures. A course of reading will be laid down, and weekly written exercises will test the work of students in following systematically and continuously the lectures and the prescribed reading. Large parts of Mill’s Principles of Political Economy will be read, as well as parts of other general books; while detailed references will be given for the reading on the application and illustration of economic principles.

Source: Harvard University. Faculty of Arts and Sciences. Division of History and Political Science Comprising the Departments of History and Government and Economics, 1897-98,  p. 31.

Economics 1: Enrollment

[Economics] 1. Professor [Frank William] Taussig, Asst. Professor [Edward] Cummings, Dr. [John] Cummings, and Messrs. [Charles Sumner] Griffin, [Charles Whitney] Mixter and [Edward Henry] Warren. — Outlines of Economics. — Principles of Political Economy.— Social Questions, and Financial Legislation.  3 hours.

Total 381: 32 Seniors, 99 Juniors, 199 Sophomores, 14 Freshmen, 37 Others.

Source:   Harvard University, Annual Reports of the President and Treasurer of Harvard College, 1897-98, p. 77.

1897-98
Economics 1.
[Mid-year Examination]

  1. Is a shop building, situated on a busy city street, capital? is the land on which it stands capital? Is a dwelling on a fashionable city street capital? the land on which it stands?
  2. Does the rent of a piece of land determine its price, and if so, how? or does its price determine the rent, and if so, how?
  3. Do you believe that differences in wages in different occupations would cease if, by gratuitous education and support, access to each occupation were made equally easy for all?
  4. Mention a case in which the income received by a person doing no manual labor is to be regarded as wages; one in which it is to be regarded as profits; one in which it is to be regarded as interest; and one in which the classification would be regarded as doubtful.
  5. Explain what is meant by the effective desire of accumulation; and consider whether, in a country like England, the minimum return on capital fixed by it has been reached.
  6. “The quantity demanded [of any commodity] is not a fixed quantity, even at the same time and place; it varies according to the value; if the thing is cheap, there is usually a demand for more of it than when it is dear. The demand, therefore, depends partly on the value. But it was laid down before that value depends on the demand. From this contradiction, how shall we extricate ourselves? How solve the paradox, of two things, each depending on the other?”
    What answer did Mill give to the question thus put by him?
  7. Does the proposition that value is determined by cost of production hold true of gold?
  8. Is it advantageous to a country to substitute paper money completely for specie?
  9. Trace the consequences of an issue of inconvertible paper, greater in amount than the specie previously in circulation, on prices, on the foreign exchanges, and on the relations of debtor and creditor.

Source: Harvard University Archives. Harvard University, Mid-year examinations, 1852-1943. Box 4. Bound Volume: Examination Papers, Mid-Years 1897-98.

1897-98.
Economics 1.
[Final Examination]

Arrange your answers strictly in the order of the questions.

  1. In what sense does Mill use the terms “value” and “price”? Professor Hadley? What do you conceive to be meant by the “socialistic theory” of value?
  2. “Many people regard the luxury of the rich as being on the whole a means of preventing harm to the poor. They regard free expenditure of the capitalists’ money as a gain to laborers, and its saving as a loss.” Is this view sound?
  3. What services are rendered to society by commercial speculation? by industrial speculation?
  4. Patent-laws, protection by customs duties, private ownership of land, — wherein analogous, in Professor Hadley’s view? in your own view?
  5. Does Mill regard the rent of land as an “unearned increment”? Does Professor Hadley? On what grounds do they reach their conclusions?
  6. “By far the most important form of consumers’ coöperation is exemplified in government management of industrial enterprises.” Why, or why not, is government management to be regarded as a form of consumers’ coöperation? What other forms of such coöperation have had wide development?
  7. The peculiarities of labor considered as a commodity; and the grounds on which it is concluded that “the members of trade unions are in a condition entirely like that of the sellers of other commodities.”
  8. Consider how, according to Mill, successive issues of paper-money will affect the supply of specie in a country; and explain how far this theoretical conclusion was or was not verified by the mode in which the silver currency (dollars and certificates) issued under the act of 1878 affected the supply of gold in the United States.
  9. On what ground does Mill object to the issue of inconvertible paper? On what ground does Professor Dunbar object to the legal tender paper now issued by the United States? Wherein are the objections similar, wherein different?
  10. “If we try to make things for which we have only moderate advantages, and in so doing divert labor and capital from those where we have extraordinary ones, we do not, in general, make money; we lose more than we gain.” — HADLEY. Point out wherein this statement is akin to the analysis of international trade by Mill, and explain precisely what is here meant by making or losing money.

Source: Harvard University Archives. Examination Papers, 1873-1915. Box 5, Examination Papers 1898-99, Bound Volume, pp. 40-41.

_______________________

Mediaeval Economic History of Europe
Economics 10
[Omitted in 1897-98.]

[*10. The Mediaeval Economic History of EuropeTu., Th., (and at the pleasure of the instructor) Sat., at 12. Professor [William James] Ashley.]

The object of this course is to give a general view of the economic development of society during the Middle Ages. It will deal, among others, with the following topics: — the manorial system in its relation to mediaeval agriculture and serfdom ; the merchant gilds and the beginnings of town life and of trade ; the craft gild and the gild-system of industry, compared with earlier and later forms; the commercial supremacy of the Hanseatic and Italian merchants ; the trade routes of the Middle Ages and of the sixteenth century ; the merchant adventurers and the great trading companies ; the agrarian changes of the fifteenth nd sixteenth centuries and the break-up of the mediaeval organization of social classes ; the appearance of new manufactures and of the domestic industry.

Special attention will be devoted to England, but that country will be treated as illustrating the broader features of the economic evolution of the whole of western Europe; and attention will be called to the chief peculiarities of the economic history of France, Germany, and Italy.

Students will be introduced in this course to the use of the original sources, and they will need to be able to translate easy Latin.

It is desirable that they should already possess some general acquaintance with mediaeval history, and those who are deficient in this respect will be expected to read one or two supplementary books, to be suggested by the instructor. The course is conveniently taken after, before, or in conjunction with History 9; and it will be of especial use to those who intend to study the law of Real Property.

Source: Harvard University. Faculty of Arts and Sciences. Division of History and Political Science Comprising the Departments of History and Government and Economics, 1897-98, pp. 31-32.

_______________________

Modern Economic History of Europe and America
(from 1500)
Economics 11

11. The Modern Economic History of Europe and America (from 1500)Tu., Th., (and at the pleasure of the instructor) Sat., at 12. Professor [William James] Ashley.

This course, — which will usually alternate with Course 10 in successive years, — while intended to form a sequel to Course 10, will nevertheless be independent, and may usefully be taken by those who have not followed the history of the earlier period. The main thread of connection will be found in the history of trade; but the outlines of the history of agriculture and industry will also be set forth, and the forms of social organization dependent upon them. England, as the first home of the “great industry,” will demand a large share of attention; but the parallel or divergent economic history of the United States, and of the great countries of western Europe, will be considered side by side with it.

Source: Harvard University. Faculty of Arts and Sciences. Division of History and Political Science Comprising the Departments of History and Government and Economics, 1897-98, p. 32.

 Economics 11: Enrollment
1897-98

[Economics] 11. Professor Ashley— The Modern Economic History of Europe and America (from 1500). 2 or 3 hours.

Total 16: 9 Graduates, 5 Seniors, 1 Junior, 1 Sophomore.

Source:   Harvard University, Annual Reports of the President and Treasurer of Harvard College, 1897-98, p. 77.

1897-98.
Economics 11.
[Mid-year Examination]

N.B.—Not more than eight questions should be attempted.

  1. “Locutus sum de breviori via ad loca aromatum per maritimam navigationem quam sit ea quam facitis per Guineam.” Translate and comment upon this.
  2. Give some account of the Fairs of Champagne.
  3. What light does Jones’ account of agricultural conditions in Europe in his own time cast upon the agrarian history of England in the 15th and 16th centuries? Be as definite as possible in your answer.
  4. What do you suppose happened to the “craft-gilds” of England during the reign of Edward VI?
  5. Discuss the purpose and effect of the statute 5 Eliz. c. 4, in the matter of the Assessment of Wages.
  6. What were the essential characteristics of the “Domestic System” of Industry?
  7. Give some account of the industrial legislation of France in the 16th century.
  8. “The policy of Europe occasions a very important inequality in the whole of the advantages and disadvantages of the different employments of labour and stock.” What had the writer of this passage in mind?
  9. Give some account of any four of the following: Albuquerque, Veramuyden, Jacob Fugger, John Hales, Jacques Cartier, Bartholomew Diaz, Barthelemy Laffemas.
  10. Give a critical account of any really important work, not prescribed, of which you have read any considerable portion in connection with this course.

Source: Harvard University Archives. Harvard University, Mid-year examinations, 1852-1943. Box 4. Bound Volume: Examination Papers, Mid-Years 1897-98.

1897-98.
Economics 11.
[Year-end Examination]

N.B.- Not more than eight questions should be attempted.

  1. “Publicae mendicationis licentiam posse civium legibus cohiberi ad liquidum ostendit ille absolutus Theologus, loannes Major.” Translate; and shew the significance of the position thus maintained.
  2. Illustrate from the history of Hamburg the change in the position of the Hanseatic League during the 16th century.
  3. Distinguish between the various races of immigrants into England since 1500, and state shortly the several respects in which the trade and industry of England were influenced by each.
  4. “Perhaps the wisest of all the commercial regulations of England.” Give a brief account of the enactment of which Adam Smith thus speaks; distinguish between the various aspects in which it may be regarded; and give your own opinion as to the justice of Adam Smith’s observation.
  5. Explain the position of “les Six Corps” at Paris. Does London furnish any analogous institutions?
  6. “Hitherto,” i.e. up to 1750, “industry had been chiefly carried on in England by numbers of smaller capitalists who were also manual workmen.” Criticise this as a bit of exposition.
  7. The position of Arthur Young in economic history.
  8. The commercial policy of the younger Pitt.
  9. Mention, with the briefest possible comment, some of the more important features in which the agricultural, industrial and commercial life of the England of to-day differs from that of the England of 1750.
  10. What were the principal defects in the administration of the English Poor Laws prior to 1834, and how was it sought to remedy them?
  11. Explain the need for the English Factory Acts, and give some account of their history.
  12. Give a critical estimate of any really important book, not prescribed, of which you have read any considerable proportion in connection with this course during the second half-year.

Source: Harvard University Archives. Examination Papers, 1873-1915. Box 5, Examination Papers 1898-99, Bound Volume, pp. 50-51.

_______________________

The Economic History of the United States
Economics 6

6. The Economic History of the United StatesTu., Th., at 2.30, and a third hour at the pleasure of the instructors. Mr. [Guy Stevens] Callender.

Course 6 gives a general survey of the economic history of the United States from the formation of the Union to the present time, and considers also the mode in which economic principles are illustrated by the experience so surveyed. A review is made of the financial history of the United States, including Hamilton’s financial system, the second Bank of the United States and the banking systems of the period preceding the Civil War, coinage history, the finances of the Civil War, and the banking and currency history of the period since the Civil War. The history of manufacturing industries is taken up in connection with the course of international trade and of tariff legislation, the successive tariffs being followed and their economic effects considered. The land policy of the United States is examined partly in its relation to the growth of population and the inflow of immigrants, and partly in its relation to the history of transportation, including the movement for internal improvements, the beginnings of the railway system, the land grants and subsidies, and the successive bursts of activity in railway building. Comparison will be made from time to time with the contemporary economic history of European countries.

Written work will be required of all students, and a course of reading will be prescribed, and tested by examination. The course is taken advantageously with or after History 13. While an acquaintance with economic principles is not indispensable, students are strongly advised to take the course after having taken Economics 1, or, if this be not easy to arrange, at the same time with that course.

Source: Harvard University. Faculty of Arts and Sciences. Division of History and Political Science Comprising the Departments of History and Government and Economics, 1897-98, pp. 32-33.

Economics 6: Enrollment

[Economics] 6. Dr. Callender. — The Economic History of the United States. 3 hours.

Total 94: 4 Graduates, 38 Seniors, 41 Juniors, 8 Sophomores, 1 Sophomore, 2 Others.

Source:   Harvard University, Annual Reports of the President and Treasurer of Harvard College, 1897-98, p. 78.

1897-98.
Economics 6.
[Mid-year Examination]

[Omit one question from each group]

I.

  1. “The effect of England’s policy was, through a restriction of the market, to render the production of those staple commodities (i.e. of agriculture, and the fisheries) less profitable. Thus New England, and later the middle colonies, not being allowed to exchange their normal products for England’s manufactures, were forced to begin manufacturing for themselves.”—
    “Briefly describe the measures designed to prevent the rise of manufactures in the colonies; and state whether in your opinion the growth of manufactures in the northern colonies was stopped chiefly by this legislation or by other causes.
  2. The American colonies during the Revolution were in much the same economic position as the South during the Rebellion. The chief resource of the latter was the value of its cotton crop to the world; that of the former was the supposed value of their trade to the nations of Europe. Describe the various ways in which the Revolutionary statesmen made use of this resource.
  3. Judging from the opinions of statesmen as well as from acts of legislation what would you say were the leading objects of American Commercial policy from 1783 to 1789?
  4. State briefly the exact circumstances which permitted the growth of American commerce during the years from 1793 to 1806. How did this temporary commercial prosperity affect the subsequent growth of manufactures?

II.

  1. Compare the conditions which gave rise to manufactures in the northern colonies before 1760 with those which prevailed during the years immediately following 1783 and 1815; and indicate what conclusions you would draw from such a comparison, as to the necessity or expediency of protective legislation to secure the development of manufactures in a new country.
  2. Discuss the effect of the duties on Cotton and Iron during the period from 1816 to 1833.
  3. Compare the treatment of wages in Hamilton’s Report on Manufactures with that which appeared in the debate on the Tariff Act of 1846. How do you explain the change?
  4. Mention several industries which were created or greatly promoted by inventions between 1840 and 1860.

III.

  1. Henry Clay declared in 1832 that the seven years preceding 1824 “exhibited a scene of the most widespread dismay and desolation,” while the seven years following 1824 exhibited the “greatest prosperity which this people, bare enjoyed since the establishment of the present constitution.” How do you explain this change?
  2. In what ways have the people of the United States made use of the Federal and State governments to provide transportation facilities? How do you explain this tendency to State interference in industrial affairs at so early a date in America?
  3. Describe the abuses in Railroad management which the Interstate Commerce Act was intended to correct.
  4. Explain why competition proves less effective in regulating freight rates than in regulating the price of most commodities.

Source: Harvard University Archives. Harvard University, Mid-year examinations, 1852-1943. Box 4. Bound Volume: Examination Papers, Mid-Years 1897-98.

1897-98.
Economics 6.
[Year-end Examination]

Answer at least eight questions.

  1. Give your reasons for agreeing to, or dissenting from, the following proposition: Until the wars of the French Revolution temporarily suspended the colonial policy of Continental Europe, the United States was in a more unfavorable economic position than they had been in prior to the Revolution
  2. Why was the adoption of a liberal tariff policy by the U.S. in 1846 more justifiable than in 1816?
  3. “The provisions of the constitution were universally considered as affording a complete security against the danger of paper money. The introduction of the banking system met with a strenuous opposition on various grounds; but it was not apprehended that bank notes, convertible at will into specie, and which no person could be legally compelled to take in payment, would degenerate into pure paper money, no longer paid at sight in specie… It was the catastrophe of 1814 which first disclosed not only the insecurity of the American banking system, as it then existed, but also that when a paper currency, driving away, and suspending the use of gold and silver, has insinuated itself through every channel of circulation, and become the only medium of exchange, every individual finds himself, in fact compelled to receive such currency, even when depreciated more than twenty per cent. in the same manner as if it had been a legal tender.” — GALLATIN.
    Prior to the adoption of the national banking system in 1863, how did the federal government attempt to prevent the evil here described. and with what success?
  4. How far do the conditions, which render competition ineffective as a regulator of transportation charges, prevail in any of the industries in which Trusts have been formed? — or to ask the same thing in another way, how far is it possible to justify Trusts on the same grounds as Railroad Pools?
  5. What reasons would you assign for the change in the relative value of gold and silver which occurred after 1873?
  6. What difficulty did the Treasury department encounter in administering the silver act of 1878, and what means were used to overcome it?
  7. Compare the effect of the protective duties on wool and woollens since 1867 with the effect of those on silk and steel during the same time.
  8. “In the division of employments which has taken place in America, the far preferable share, truly, has fallen to the Northern States…The states, therefore, which forbid slavery, having reaped the economical benefits of slavery, without incurring the chief of its moral evils, seem to be even more indebted to it than the slave states.” — WAKEFIELD.
    How would you explain this statement?
  9. According to Mr. Cairnes, what constituted the economic basis of Negro slavery in the Southern States and enabled it to successfully resist the competition of free white labor? Do you consider this economic basis of slavery to have been permanent?
  10. Describe the most important change in Southern agrarian conditions which has resulted from emancipation.
  11. What influences can you mention that have contributed to the fall in the prices of the staple products of Northern agriculture during the last ten years?
  12. Why has this fall in the price of agricultural products caused greater hardship to the farmers than the corresponding fall in the price of manufactured products has caused among manufacturers?

Source: Harvard University Archives. Examination Papers, 1873-1915. Box 5, Examination Papers 1898-99, Bound Volume, pp. 45-47.

_______________________

History and Literature of Economics to the Close of the 18th Century
Economics 15

*15. The History and Literature of Economics to the Close of the Eighteenth CenturyMon., Wed., (and at the pleasure of the instructor) Fri., at 12.     Professor[William James] Ashley.

The course of economic speculation will here be followed, in its relation alike to the general movement of contemporary thought and to contemporary social conditions. The lectures will consider the economic theories of Plato and Aristotle; the economic ideas underlying Roman law; the mediaeval church and the canonist doctrine; mercantilism in its diverse forms; “political arithmetic;” the origin of the belief in natural rights and its influence on economic thought; the physiocratic doctrine; the work and influence of Adam Smith; the doctrine of population as presented by Malthus; Say and the Erench school; and the beginnings of academic instruction in economics.

The lectures will be interrupted from time to time for the examination of selected portions of particular authors; and careful study will be given to portions of Plato’s Republic and Aristotle’s Politics (in translation) to Mun’s England’s Treasure, Locke’s Considerations of the Consequences of the Lowering of Interest, certain Essays of Hume, Turgot’s Réflexions, and specified chapters of Adam Smith’s Wealth of Nations, and Malthus’ Essay. Students taking the course are expected to procure the texts of the chief authors considered, and to consult the following critical works:

Ingram, History of Political Economy; Cossa, Introduction to the Study of Political Economy; Cannan, History of the Theories of Production and Distribution; Bonar, Philosophy and Political Economy; Böhm-Bawerk, Capital and Interest; Taussig, Wages and Capital.

Source: Harvard University. Faculty of Arts and Sciences. Division of History and Political Science Comprising the Departments of History and Government and Economics, 1897-98, pp. 33-34.

Economics 15: Enrollment
1897-98

[Economics] 15. Professor Ashley. — The History and Literature of Economics to the Middle of the Nineteenth Century. 2 or 3 hours.

Total 6: 3 Graduates, 1 Senior, 2 Sophomores.

Source:   Harvard University, Annual Reports of the President and Treasurer of Harvard College, 1897-98, p. 77.

1897-98.
Economics 15.
[Mid-year Examination]

N.B.—Not more than eight questions should be attempted.

  1. Compare the Republic of Plato with the Politics of Aristotle, as to purpose and temper.
  2. Expound Aristotle’s teaching with regard to Slavery.
  3. “He is supposed to have given a striking proof of his wisdom, but his device for getting money is of universal application.” Comment, and explain the context.
  4. What parts of Aristotle’s criticism of Communism seem to you pertinent to modern Socialism. Explain what particular kind of Socialism you have in mind.
  5. Set forth, and criticise, Maine’s account of the influence in modern times of the conception of a Law of Nature.
  6. Were the early Christians communists?
  7. How did “Inter-est [sic],” in its original meaning, differ from “Usury.”
  8. The position in economic literature of Nicholas Oresme.
  9. What principles, if any, of the canonist teaching seem to you to have any bearing on modern economic problems.
  10. What were “the particular ways and means to encrease our exportations and diminish our importations,” according to Mun?

Source: Harvard University Archives. Harvard University, Mid-year examinations, 1852-1943. Box 4. Bound Volume: Examination Papers, Mid-Years 1897-98.

1897-98.
Economics 15.
[Year-end Examination]

  1. Criticise the current conception of “Mercantilism” in the light of your own study of the later English mercantilist writers.
  2. The place of Locke in English economic thought.
  3. “Tout ce qu’il y a de vrai dans ce volume estimable, mais pénible à lire, en deux gros volumes in-4°, se trouve dans les Réflexions de Turgot; tout ce qu’Adam Smith y a ajouté manque d’exactitude et même de fondement.”
    Translate, and then criticise this remark of Du Pont’s.
  4. Trace the various elements which went to make up the idea of Nature in Adam Smith’s mind, and then explain Smith’s application of it to any particular subject.

Source: Harvard University Archives. Examination Papers, 1873-1915. Box 5, Examination Papers 1898-99, Bound Volume, p. 53.

_______________________

Economic Theory
in the 19th Century
Economics 2

*2. Economic Theory in the Nineteenth CenturyMon., Wed., Fri., at 2.30. Professor [Frank William] Taussig.

Course 2 is designed to acquaint the student with the history of economic thought during the nineteenth century, and to give him at the same time training in the critical consideration of economic principles. The exercises are accordingly conducted mainly by the discussion of selected passages from the important writers; and in this discussion students are expected to take an active part. Lectures are given at intervals, tracing the general movement of economic thought and describing its literature. Special attention will be given to the theory of distribution.

The course opens with an examination of Ricardo’s doctrines, selections from Ricardo’s writings being read and discussed. These will then be compared with the appropriate chapters in Mill’s Principles of Political Economy, and further with passages in Cairnes’ Leading Principles. The theory of wages, and the related theory of business profits, will then be followed in the writings of F. A. Walker, Sidgwick, and Marshall, and a general survey made of the present stage of economic theory in England and the United States. The development on the continent of Europe will be traced chiefly in lectures; but toward the close of the year a critical examination will be made of the doctrines of the modern Austrian school.

Course 2 is taken with advantage in the next year after Course 1; but Course 15 may also be taken with advantage after Course 1, and then followed by Course 2, or taken contemporaneously with it.

Source:  Harvard University. Faculty of Arts and Sciences. Division of History and Political Science Comprising the Departments of History and Government and Economics, 1897-98, p. 34.

 Economics 2: Enrollment
1897-98

[Economics] 2. Professor Taussig. — Economic Theory in the Nineteenth Century. 3 hours.

Total 32: 9 Graduates, 9 Seniors, 11 Juniors, 3 Sophomores.

Source:   Harvard University, Annual Reports of the President and Treasurer of Harvard College, 1897-98, p. 77.

1897-98.
Economics 2.
[Mid-year. 1898.]

[Arrange your answers in the order of the questions. One question may be omitted.]

  1. According to Ricardo, what is the effect, if any, of a rise in the price of food on wages? on profits? on the prices of commodities?
  2. “Ricardo expresses himself as if the quantity of labor which it costs to produce a commodity and bring it to the market, were the only thing on which its value depended. But since the cost of production to the capitalist is not labor but wages, and since wages may be greater or less, the quantity of labor being the same; it would seem that the value of the product cannot be determined solely by the quantity of labor, but by the quantity together with the remuneration; and that values must depend on wages.” — Mill.
    What do you conceive Ricardo would have said to this?
  3. “We have therefore remarked that the difficulty of passing from one class of employments to a class greatly superior, has hitherto caused the wages of all those classes of laborers who are separated from one another by any very marked barrier, to depend more than might be supposed upon the increase of population of each class, considered separately; and that the inequalities in the remuneration of labor are much greater than could exist if the competition of the laboring people generally could be brought practically to bear on each particular employment. It follows from this that wages in each particular employment do not rise or fall simultaneously, but are, for short and sometimes even for long periods, nearly independent of each other. All such disparities evidently alter the relative costs of production of different commodities, and will therefore be completely represented in the natural or average value.” — Mill.
    What has Cairnes added to this?
  4. “He [Mr. Longe] puts the case of a capitalist who, by taking advantage of the necessities of his workmen, effects a reduction in their wages; and asks how is this sum, thus withdrawn, to be restored to the fund? . . . The answer to the case put by Mr. Longe is easy on his own principles; and I am disposed to flatter myself that the reader who has gone with me in the foregoing discussion will not have much difficulty in replying to it on mine.” — Cairnes.
    Give the reply.
  5. “Fixity or definiteness is the very essence of the supposed wages-fund. No one denies that some amount or other must within a given period be disbursed in the form of wages. The only question is whether that amount be determinate or indeterminate.” — Thornton.
    What is Cairnes’s answer to the question put in this passage?
  6. What would you expect the relation of imports to exports to be in a country whose inhabitants had for a long time been borrowing, and were still borrowing, from the inhabitants of other countries?
  7. Are general high wages an obstacle to a country’s exporting?
  8. “Granted a certain store of provisions, of tools, and of materials for production, sufficient, say, for 1000 laborers, those who hold the wage-fund theory assert that the same rate of wages (meaning thereby the actual amount of necessaries, comforts, and luxuries received by the laborer) would prevail whether these laborers be Englishmen or East Indians. . . . On the contrary, it is not true that the present economical quality of the laborers, as a whole, is an element in ascertaining the aggregate amount that can now be paid in wages; that as wages are paid out of the product, and as the product will be greater or smaller by reason of the workman’s sobriety, industry, and intelligence, or his want of these qualities, so wages may and should be higher or lower accordingly?”
    Give your opinion.
  9. What do you conceive to be the “no profits class of employers” in President Walker’s theory of distribution?

Source: Harvard University Archives. Harvard University, Mid-year examinations, 1852-1943. Box 4. Bound Volume: Examination Papers, Mid-Years 1897-98.

1897-98.
Economics 2.
[Final Examination]

The answer to one question may be omitted.

  1. The analysis of capital in its relation to labor and wages at the hands of Ricardo and of Böhm-Bawerk, — wherein the same? wherein different?
  2. The contributions of permanent worth for economic theory by Cairnes? by F.A. Walker? [Consider one.]
  3. The position of Carey and Bastiat in the development of economic theory.
  4. “If the efficiency of labor could be suddenly doubled, whilst the capital of the country remained stationary, there would be a great and immediate rise in real wages. The supplies of capital already in existence would be distributed among the laborers more rapidly than would otherwise be the case, and the increased efficiency of labor would soon make good the diminished supplies. The fact is that an increase in the efficiency of labor would bring about an increase in the supply of capital.” — Marshall. Why? or why not?
  5. “The capital of the employer is by no means the real source of the wages even of the workmen employed by him. It is only the intermediate reservoir from which wages are paid out, until the purchasers of the commodities produced by that labor make good the advance and thereby encourage the undertaker to purchase additional labor.” W. Roscher.
    What do you say to this?
  6. “If the rate of profit falls, the laborer gets more nearly the whole amount of the product. But if the rate of wages falls, we have a corresponding fall in prices and little change in the relative shares of labor and capital.” Hadley.
    Why, or why not, in either case?
  7. “In the present condition of industry, most sales are made by men who are producers or merchants by profession, and who hold an amount of commodities entirely beyond any needs of their own. Consequently, for them the subjective use-value of their own wares is, for the most part, very nearly nil; and the figure which they put on their own valuation almost sinks to zero.” Explain the bearing of this remark on the theory of value as developed by Böhm-Bawerk.
  8. What, according to Böhm-Bawerk, is the explanation of interest derived from “durable consumption goods”? And what is your own view?
  9. How far do you conceive that there is a “productivity” of capital, serving to explain the existence of interest, and the rate?

Source: Harvard University Archives. Examination Papers, 1873-1915. Box 5, Examination Papers 1898-99, Bound Volume, pp. 41-42.

_______________________

Scope and Method in Economic Theory and Investigation
Economics 132

*132 hf. Scope and Method in Economic Theory and Investigation. Half-course (second half-year). Mon., Wed., Fri., at 11. Professor [William James] Ashley.

Course 13 will examine the methods by which the important writers, from Adam Smith to the present time, have approached economic questions, and the range which they have given their inquiries; and will consider the advantage of different methods, and the expediency of a wider or narrower scope of investigation. Mill’s essay on the Definition of Political Economy; Cairnes’ Logical Method of Political Economy; Keynes’ Scope and Method of Political Economy; certain sections of Wagner’s Grundlegung and Schmoller’s essay on Volkswirthschaft will be carefully examined. The conscious consideration of method by the later writers of the classic school and by their successors in England; the rise of the historical school and its influence; the mode in which contemporary writers approach the subject, — will he successively followed.

Course 13 is open to students who take or have taken Course 2 or Course 15. A fair reading knowledge of German as well as of French will be expected of students, and the opportunity will be taken to assist them to acquire facility in reading scientific German. Subjects will be assigned for investigation and report, and the results of such investigations will be presented for discussion.

Source: Harvard University. Faculty of Arts and Sciences. Division of History and Political Science Comprising the Departments of History and Government and Economics, 1897-98, pp. 34-35.

Economics 132: Enrollment
1897-98

[Economics] 132. Professor Ashley. — Scope and Method in Economic Theory and Investigation. 3 hours.

Total 5: 3 Graduates, 1Senior, 1 Sophomore.

Source:   Harvard University, Annual Reports of the President and Treasurer of Harvard College, 1897-98, p. 77.

1897-98.
Economics 132.
[Year-end Examination]

  1. “Ganz unabhängig von der deutschen historischen National-Ökonomie haben Sociologen wie A. Comte ähnliche, freilich auch zu weit gehende Bedenken gegen Deduction und Abstraction der britischen Oekonomik erhoben.”
    Translate this; and then (1) state Comte’s position with regard to economic method, (2) criticise it.
  2. “Die besondere Leistung des wissenschaftlichen Socialismus ist der Nachweis des beherrschenden Einflusses der Privateigenthums ordnung, speciell des Privateigenthums‚ an den sachlichen Productionsmitteln’, auf die Gestaltung der Production und der Vertheilung des Productionsertrag, zumal bei Wegfall aller Beschränkungen der Verfügungsbefugnisse des Privateigenthümers im System der freien Concurrenz…Durch den Socialismus ist aber auch das andere grosse Hauptproblem, dasjenige der Freiheit und ihrer Rechtsordnung, in ein neues Stadium getreten. Hier begeht der Socialismus nun jedoch trotz seiner scharfen Kritik der wirthschaftlichen Freiheit im System der ökonomischen Individualismus und Liberalismus principiell denselben Fehler, wie letzterer: auch er fasst die Freiheit als Axiom, statt als Problem auf, ein schwerstes Problem gerade jeder socialistischen Rechts- und Wirtschaftsordnung.”
    (1) Translate, (2) explain, and (3) comment on this.
  3. Discuss the questions raised by the application to Economies of the distinction between a Science and an Art.
  4. What did J. S. Mill mean by the Historical Method? Consider (1) the source of the idea, (2) its characterization by Mill, and (3) the bearing of his utterances with regard to it upon the question of economic method.
  5. Examine either (1) the Malthusian doctrine of Population or (2) the Ricardian doctrine of Rent as a specimen of an economic “law.”

Source: Harvard University Archives. Examination Papers, 1873-1915. Box 5, Examination Papers 1898-99, Bound Volume, pp. 52-53.

_______________________

Principles of Sociology
Economics 3

*3. The Principles of Sociology. Mon., Wed., and (at the pleasure of the instructor) Fri., at 1.30. Asst. Professor Edward Cummings.

Course 3 begins with a general survey of the structure and development of society; showing the changing elements of which a progressive society is composed, the forces which manifest themselves at different stages in the transition from primitive conditions to complex phases of civilized life, and the structural outlines upon which successive phases of social, political, and industrial organization proceed. Following this, is an examination of the historical aspects which this evolution has actually assumed: Primitive man, elementary forms of association, the various forms of family organization, and the contributions which family, clan and tribe have made to the constitution of more comprehensive ethnical and political groups ; the functions of the State, the circumstances which determine types of political association, the corresponding expansion of social consciousness, and the relative importance of military, economic, and ethical ideas at successive stages of civilization. Special attention is given to the attempts to formulate physical and psychological laws of social growth; to the relative importance of natural and of artificial selection in social development; the law of social survival; the dangers which threaten civilization; and the bearing of such general consideration upon the practical problems of vice, crime, poverty, pauperism, and upon mooted methods of social reform.

The student is thus acquainted with the main schools of sociological thought, and opportunity is given for a critical comparison of earlier phases of sociological theory with more recent contributions in Europe and the United States. Regular and systematic reading is essential. Topics are assigned for special investigation in connection with practical or theoretical aspects of the course.

Source: Harvard University. Faculty of Arts and Sciences. Division of History and Political Science Comprising the Departments of History and Government and Economics, 1897-98, p. 35.

 Economics 3: Enrollment
1897-98

[Economics] 3 Asst. Professor E. Cummings. — The Principles of Sociology. — Development of the Modern State, and of its Social Functions. 2 or 3 hours.

Total 59: 4 Graduates, 30 Seniors, 13 Juniors, 6 Sophomores, 6 Others.

Source:   Harvard University, Annual Reports of the President and Treasurer of Harvard College, 1897-98, p. 77.

1897-98.
Economics 3.
[Mid-year examination]

[Answer the questions in the order in which they stand. Give one hour to each group.]

I.

Discuss the merits and defects of the following conceptions of society:

A) Society as an organism.
B) Society as a physio-psychic organism.
C) Society as an organization.
D) Society as an “organisme contractuel.”

What in your opinion are the essential differences between an ant hill and a human society?

II.

Give a critical summary and comparison of the views of Spencer, Giddings, Ritchie in regard to (a) the origin, (b) the development and forms, and (c) the functions of political organization.
Contrast the ancient, medieval and modern views of the relations of the State to Society and to the Individual.

III.

Discuss the views of Spencer, Westermarck, Giddings and others on the causes and the effects of the successive phases of family organization.
What claims has the family to be regarded as the “social unit”?
Discuss the significance of existing tendencies.

Source: Harvard University Archives. Harvard University, Mid-year examinations, 1852-1943. Box 4. Bound Volume: Examination Papers, Mid-Years 1897-98.

1897-98.
Economics 3.
[Final Examination]

I.

The nature, the causes, and the criteria of progress, according to (a) Spencer, (b) Kidd, (c) La Pouge, (d) Haycraft, (e) Giddings, (f) Tarde? State and illustrate by historical examples your own views in regard to the “curve of progress.”

II.

“The special feature of the final adjustment secured by our occidental civilizations, contrary to what has been seen on the earth before them, will therefore have been the subordination of the social to the individual. This singularly daring enterprise is the true novelty of modern times. It is well worth living to second it or to participate in it.”— TARDE.
“There seems no avoiding the conclusion that these conspiring causes must presently bring about that lapse of self-ownership into ownership by the community, which is partially implied by collectivism and completely by communism.” — SPENCER.
Discuss carefully the merits of these opinions, and the evidence on which they rest.

III.

What do you conceive to be some of the dangerous tendencies of our civilization? And what are the remedies?

IV.

State the subject of your final report and the reading you have done in connection with it.

Source: Harvard University Archives. Examination Papers, 1873-1915. Box 5, Examination Papers 1898-99, Bound Volume, p. 42.

_______________________

Socialism and Communism
Economics 14

*14. Socialism and Communism. — History and Literature. Tu., Th., and at the pleasure of the instructor) Sat., at 9. Asst. Professor Edward Cummings.

Course 14 is primarily an historical and critical study of socialism and communism. It traces the history and significance of schemes for social reconstruction from the earliest times to the present day. It discusses the historical evidences of primitive communism, the forms assumed by private ownership at different stages of civilization, the bearing of these considerations upon the claims of modern socialism, and the outcome of experimental communities in which socialism and communism have actually been tried. Special attention, however, is devoted to the recent history of socialism, — the precursors and the followers of Marx and Lassalle, the economic and political programmes of socialistic parties in Germany, France, and other countries.

The primary object is in every case to trace the relation of historical evolution to these programmes; to discover how far they have modified history or found expression in the policy of parties or statesmen; how far they must be regarded simply as protests against existing phases of social evolution; and how far they may be said to embody a sane philosophy of social and political organization.

The criticism and analysis of these schemes gives opportunity for discussing from different points of view the ethical and historical value of social and political institutions, the relation of the State to the individual, the political and economic bearing of current socialistic theories.

The work is especially adapted to students who have had some introductory training in Ethics as well as in Economics. A systematic course of reading covers the authors discussed; and special topics for investigation may be assigned in connection with this reading.

Source: Harvard University. Faculty of Arts and Sciences. Division of History and Political Science Comprising the Departments of History and Government and Economics, 1897-98, pp. 35-36.

Economics 14: Enrollment

[Economics] 14. Asst. Professor E. Cummings. — Communism and Socialism. — History and Literature. 2 or 3 hours.

Total 12: 3 Graduates, 5 Seniors, 2 Juniors, 2 Sophomores.

Source: Harvard University. Report of the President of Harvard College, 1897-98, p. 78.

1897-98.
Economics 14.
[Mid-year Examination]

Outline briefly the characteristics of socialistic theory and practice in ancient, medieval and modern times, — devoting about an hour to each epoch, and showing —

(a) so far as possible the continuity of such speculations; the characteristic resemblances and differences;
(b) the influence of peculiar historical conditions;
(c) the corresponding changes in economic theory and practice.

Source: Harvard University Archives. Harvard University, Mid-year examinations, 1852-1943. Box 4. Bound Volume: Examination Papers, Mid-Years 1897-98.

No Year-end Examination for 1898 found.

_______________________

Labor Question
in Europe and the U.S.
Economics 9.

9. The Labor Question in Europe and the United States. — The Social and Economic Condition of Workingmen. Tu., Th., Sat., at 10. Asst. Professor Edward Cummings and Dr. John Cummings.

Course 9 is a comparative study of the condition and environments of workingmen in the United States and European countries. It is chiefly concerned with problems growing out of the relations of labor and capital. There is careful study of the voluntarily organizations of labor, — trade unions, friendly societies, and the various forms of cooperation; of profit-sharing, sliding scales, and joint standing committees for the settlement of disputes; of factory legislation, employers’ liability, the legal status of laborers and labor organizations, state courts of arbitration, and compulsory government insurance against the exigencies of sickness, accident, and old age. All these expedients, together with the phenomena of international migration, the questions of a shorter working day and convict labor, are discussed in the light of experience and of economic theory, with a view to determining the merits, defects, and possibilities of existing movements.

The descriptive and theoretical aspects of the course are supplemented by statistical evidence in regard to wages, prices, standards of living, and the social condition of labor in different countries.

Topics will be assigned for special investigation, and students will be expected to participate in the discussion of selections from authors recommended for a systematic course of reading.

The course is open not only for students who have taken Course 1, but to Juniors and Seniors of good rank who are taking Course 1.

Source: Harvard University. Faculty of Arts and Sciences. Division of History and Political Science Comprising the Departments of History and Government and Economics, 1897-98, pp. 36-37.

Economics 9: Enrollment

[Economics] 9. Asst. Professor E. Cummings and Dr. J. Cummings. — The Labor Question in Europe and the United States. — The Social and Economic Condition of Workingmen. 3 hours.

Total 108: 1 Graduate, 39 Seniors, 51 Juniors, 12 Sophomores, 5 Others.

Source: Harvard University. Report of the President of Harvard College, 1897-98, p. 78.

No Mid-year Examination found.

1897-98.
Economics 9.
[Year-End Examination]

I.
WORKINGMEN’S INSURANCE.

“After a preliminary examination of the various kinds of working-men’s insurance, and the chief methods by which its provision can be accomplished, we have considered the history and present condition of the problem in each of the great countries of Europe and in the United States. It now remains to pass in review the whole field, to contrast, in a measure, the various policies that have been pursued, and to indicate some of the ways in which this rich experience can be of assistance in any attempt that may be made in this country to further similar movements.”
Devote one hour (a) to analyzing the present condition in each country; (b) to indicating the ways in which this rich experience can be of assistance.

II.

a) Give the name, the size, the characteristics of the important labor organizations in the United States.

b) Compare the development and present condition of labor organizations in the United States, with the movement in England.

c) How do you account for the differences in success attending trade union and coöperative enterprises in the two countries?

III.

a) What agencies, public and private, are available for settling disputes between employers and employed in the United States?

b) To what important legal questions have these disputes given rise? What has been the attitude of the judiciary and what are the merits of the present controversy in regard to injunctions?

c) What has been the general character and value of labor legislation during the last decade?

IV.

Indicate approximately the husband’s earnings, the family income and the standard of living among laborers in coal, iron, steel, textile or other industries,

1) in the United States.
2) in European countries.
3) Compare the native with the foreign-born American in these respects.
4) What conclusions do you draw from the evidence?

V.

What is the subject of your special report? State briefly (a) the method of your research, (b) the conclusions reached.

Source: Harvard University Archives. Examination Papers, 1873-1915. Box 5, Examination Papers 1898-99, Bound Volume, pp. 49-50.

_______________________

Statistics
Economics 4

*4. Statistics. — Applications to Social and Economic Problems. — Studies in Movements of Population. — Theory and Method. Mon., Wed., Fri., at 11. Dr. John Cummings.

This course deals with statistical methods used in the observation and analysis of social conditions, with the purpose of showing the relation of statistical studies to Economics and Sociology, and the scope of statistical inductions. It undertakes an examination of the views entertained by various writers regarding the theory and use of statistics, and an historical and descriptive examination of the practical methods of carrying out statistical investigations. The application of statistical methods is illustrated by studies in political, fiscal, and vital statistics, in the increase and migration of population, the growth of cities, the care of criminals and paupers, the accumulation of capital, and the production and distribution of wealth.

Source: Harvard University. Faculty of Arts and Sciences. Division of History and Political Science Comprising the Departments of History and Government and Economics, 1897-98, p. 37.

Economics 4: Enrollment
1897-98

[Economics] 4. Dr. J. Cummings. — Statistics. — Applications to Economic and Social Questions. — Studies in the Movement of Population. — Theory and Method. 3 hours.

Total 18: 7 Seniors, 7 Juniors, 3 Sophomores, 1 Other.

Source: Harvard University. Report of the President of Harvard College, 1897-1898, p. 78.

1897-98.
Economics 4.
[Mid-year Examination]

[Divide your time equally between A. and B.]

A.

[Take two.]

  1. In what sense do you understand Quetelet’s assertion that “the budget of crime is an annual taxation paid with more preciseness than any other”?
    Comment upon the “element of fixity in criminal sociology.”
    What are the “three factors of crime”?
    Can you account for the “steadiness of the graver forms of crime”? for the increase or decrease of other crimes?
    Define “penal substitutes.”
    What determines the rate of criminality?
    Comment upon the tables relating to crime in the last federal census, and explain how far they enable one to estimate the amount of crime committed and the increase or decrease in that amount.
  2. Comment upon the movement of population in the U. S. as indicated in the census rates of mortality and immigration. Upon the movement of population in France and in other European countries during this century. Can you account for the decline in the rates of mortality which characterize these populations?
    Give an account of the growth of some of the large European cities and of the migratory movements of their populations.
    Give an account for the depopulation of rural districts which has taken place during this century?
  3. Give some account of the Descriptive School of Statisticians and of the School of Political Arithmetic.
    Of the organization and work of statistical bureaus in European countries during this century.
    Of the census bureau in the United States.

B.

[Take four.]

  1. What are some of the “positive” statistical evidences of vitality in a population? “negative”?
  2. Define “index of mortality.”
  3. Comment upon the density and distribution of population in the United States.
  4. What do you understand by “normal distribution of a population according to sex and age”? Define “movement of population.”
  5. Explain the various methods of estimating a population during intercensal years.

Source: Harvard University Archives. Harvard University, Mid-year examinations, 1852-1943. Box 4. Bound Volume: Examination Papers, Mid-Years 1897-98.

1897-98.
Economics 4.
[Final Examination]

A.

I.

“The wealth of a nation is a matter of estimate only. Certain of its elements are susceptible of being approximated more closely than others; but few of them can be given with greater certainty or accuracy than is expressed in the word ‘estimated.’” Why? State the several methods used for determining the wealth of a nation. Give some account of the increase and of the present distribution of wealth in the United States.

II.

What statistical data indicate the movement of real wages during this century? What facts have to be taken into account in determining statistically the condition of wage earners? State the several methods of calculating index numbers of wages and prices, and explain the merits of each method. Explain the use of weighted averages as indexes, and the considerations determining the weights. What has been the movement of wages and prices in the United States since 1860?

III.

Statistical data establishing a hierarchy of European races, the fundamental “laws of anthropo-sociology,” and the selective influences of migratory movements and the growth of cities.

B.

Take six.

  1. “I have striven with the help of biology, statistics and political economy to formulate what I consider to be the true law of population.” (Nitti.) What is this law? Is it the true law? Why?
  2. Upon what facts rests the assertion that “the fulcrum of the world’s balance of power has shifted from the West to the East, from the Mediterranean to the Pacific”?
  3. What factors determine the rate of suicide? Consider the effect upon the rate of suicide of the sex and age distribution of the population, of the social and physical environment, and of heredity.
  4. Statistical determination of labor efficiency, and the increase of such efficiency during this century.
  5. How far are statistics concerning the number of criminal offenders indicative of the amount of criminality? Statistics of prison populations? Of crimes? What variables enter in to determine the “rate of criminality”? What significance do you attach to such rates?
  6. The statistical method.
  7. Graphics as means of presenting statistical data.

Source: Harvard University Archives. Examination Papers, 1873-1915. Box 5, Examination Papers 1898-99, Bound Volume, pp. 43-44.

_______________________

Railways and other Public Works
Economics 52

52 hf. Railways and other Public Works, under Government and Corporate management. Half-course (second half-year). Tu., Th., and (at the pleasure of the instructor) Sat., at 1.30. Mr. [Hugo Richard] Meyer.

In this course it is proposed to review the history and working of different modes of dealing with railway transportation, and to deal summarily with other similar industries, such as the telegraph, street railways, water and gas supply. Consideration will be given to the economic characteristics of these industries, the theory and history of railway rates, the effects of railway service and railway charges on other industries, the causes and consequences of monopoly conditions. The history of legislation in the more important European countries will be followed, as well as the different modes in which they have undertaken the regulation and control of private corporations, or have assumed direct ownership, with or without management and operation. Some attention will be given also to the experience of the British colonies, and more especially of those in Australia. In the United States, there will be consideration of the growth of the great systems, the course of legislation by the federal government, the working of the Interstate Commerce Act, and the modes of regulation, through legislation and through Commissions, at the hands of the several States. So far as time permits, other industries, analogous to railways, will be discussed in a similar manner.

Written work, in the preparation of papers on assigned topics, will be expected of all students in the course.

Source: Harvard University. Faculty of Arts and Sciences. Division of History and Political Science Comprising the Departments of History and Government and Economics, 1897-98, pp. 37-38.

Economics 52: Enrollment
1897-98

[Economics] 52. Mr. Meyer. — Public Works, Railways, Postal and Telegraph Service, and Monopolized Industries, under Corporate and Public Management. Hf. 3 hours. 2d half year.

Total 65: 31 Seniors, 16 Juniors, 8 Sophomores, 10 Others.

Source: Harvard University. Report of the President of Harvard College, 1897-1898, p. 78.

1897-1898.
Economics 52.
[Final Examination]

  1. “The principle [of railway rates] commonly advocated by the antagonists of the railways, as well as by the would-be reformers, is that of cost of service. Charges should be regulated in accordance with the cost of the particular transaction to the company. This is certainly not the actual method. Is it the correct method?”
    Give your reasons for accepting or rejecting the “cost of service” principle.
  2. What were the causes of the so-called granger agitation of 1871-74; of the reappearance of this agitation in 1886-88?
  3. What were the principal reasons for the instability of railway pools in the United States?
  4. By what means did the Trunk Line Associations which succeeded the Trunk Line Pool seek to limit competition and attain the effects of pooling?
  5. Discuss the working of the Interstate Commerce Act under the following headings:—
    The prohibition of undue or unreasonable preference or advantage and the prohibition of pooling.
    The construction by the United States Courts of the clause that the findings of the Commission shall be prima facie evidence in judicial proceedings.
    Legal embarrassments and other obstacles encountered by the Commission in obtaining testimony in penal cases.
    The attitude of the railways to the Act.
  6. The history of the application of the long and short haul clause to competitive rates made by railways not subject to competition from railways which are beyond the jurisdiction of the Interstate Commerce Commission; and to rates on imported commodities. Discuss under the following heading:—
    “The construction put upon the long and short haul clause by the Interstate Commerce Commission; by the United States Supreme Court.
  7. Discuss the working of the German legislation prescribing for distances over 100 km a uniform rate per ton per kilometer.
    Should you expect the practice of equal mileage charges to work with more friction or with less in the United States than in Germany?
    Alternative:
    The important points of difference between the management of the Prussian State Railways and the management of the Australian State Railways; between the management of the English Railways and the management of the American Railways.
  8. The reasons for the failure of the De Freycinet (1879) railway construction schemes; and the effect upon the French Budget of the “agreements” negotiated in 1883 between the French Government and the Six Companies.
    Alternative:
    The effect upon the Italian Budget of the “conventions” made in 1885 between the Italian Government and the Three Companies. The effect upon the Italian Exchequer of the railway construction carried out under the act of 1879 and the supplementary acts of 1881, 1882, and 1885.

Source: Harvard University Archives. Examination Papers, 1873-1915. Box 5, Examination Papers 1898-99, Bound Volume, pp. 44-45.

_______________________

Theory and Methods of Taxation
Economics 71

*71 hf. The Theory and Methods of Taxation, with special reference to local taxation in the United States. Half-course (first half-year). Tu., Th., and (at the pleasure of the instructor) Sat., at 1.30. Professor [Frank William] Taussig.

Course 71 undertakes an examination of the theory of taxation, based upon the comparative study of methods as practised in different countries and in different States of the American Union. This examination necessarily includes some discussion of leading questions in revenue legislation, such as the taxation of incomes and personal property, the single tax, progressive taxation, and indirect taxes.

Source: Harvard University. Faculty of Arts and Sciences. Division of History and Political Science Comprising the Departments of History and Government and Economics, 1897-98, p. 38.

Economics 71: Enrollment

[Economics ] 71. Professor Taussig.—The Theory and Methods of Taxation, with special reference to Local Taxation in the United States. 2 or 3 hours. 1st half year.

Total 42: 5 Graduates, 27 Seniors, 9 Juniors, 1 Sophomore.

Source: Annual Reports of the President and the Treasurer of Harvard College 1897-98, (Cambridge, Massachusetts, 1899), p. 78.

Economics 71.
Readings

Seligman—Essays in Taxation.
Bastable—Public Finance.
Leroy-Beaulieu—Science des Finances, Vol. I.
Say—Dictionnaire des Finances.
Quarterly Journal of Economics, cited as Q. J. E.
Dowell—History of Taxation in England.

PRELIMINARY QUESTIONS: CLASSIFICATION.

Seligman, Ch. IX.
Bastable, Bk. II, Ch. I; Bk. III, Ch. 1

TAXES ON LAND.

{Leroy-Beaulieu. Bk. II, Ch. VI;
Say, article “Foncière (Contribution).” 233-241.}
Bastable, Bk. IV, Ch. I.
Dictionary of Political Economy, article “Land Tax.”

HABITATION TAXES.

{Leroy-Beaulieu, Bk. II, Ch. VII.
Say, article “Personelle-Mobilière,” 850-857.}
Dowell, Vol. III, 186-192.

INCOME TAXES.

Leroy-Beaulieu, Bk. II, Ch. X.
Bastable, Bk. IV, Ch. IV.
{Dowell, Vol. III, 99-122;
Article “Income Tax in the United Kingdom,” in Dictionary of Political Economy, Vol. II.}
J. A. Hill, The Prussian Income Tax, Q. J. E., January, 1892.
Seligman, Ch. X, iii, iv.

BUSINESS TAXES.

{Say,  article “Patentes,” pp. 743-752;
Leroy-Beaulieu, Bk. II, Ch. VIII.}
J. A. Hill—The Prussian Business Tax, Q. J. E., October, 1893.

SUCCESSION TAXES.

Seligman, Ch. V; Ch. IX, i.
Bastable, Bk. III, Ch. III.

PROGRESSION.

{Leroy-Beaulieu, Bk. II, Ch. II;
Bastable, Bk. III, Ch. III.}
Seligman, Progressive Taxation, pp. 190-200; pp. 39-53 (Switzerland).

DIRECT TAXES BY THE UNITED STATES.

C. F. Dunbar,The Direct Tax of 1861, Q. J. E., July, 1889; Vol. III, pp. 436-446.
J. A. Hill,The Civil War Income Tax, Q. J. E., July, 1894.
C. F. Dunbar, The New Income Tax, Q. J. E., October, 1894.

LOCAL TAXES IN ENGLAND.

Blunden, Local Taxation and Finance, Ch. III, IV, V.

LOCAL TAXATION IN THE UNITED STATES.

Seligman, Ch. II, IV, VI, XI.
Ely, Taxation in American States, part III, Ch. VII.
Plehn, The General Property in California, (Economic Studies, Vol. II, No. 3), Part II, 151-178.
Angell, The Tax Inquisitor System in Ohio, in Yale Review, February, 1897.

Source: Harvard University Archives. Syllabi, course outlines and reading lists in Economics, 1895-2003 (HUC 8522.2.1) Box 1, folder “1897-1898”.

1897-98.
Economics 71.
[Mid-year Examination]

[Arrange your answers strictly in the order of the questions. Give and answer, however brief, to each question]

  1. Consider which of the following combinations, if any, bring about “double taxation”: (1) the impôt sur la propriété batie and the personelle-mobilière, in France; (2) local rates and schedule A of the income tax, in Great Britain; (3) the taxation of mortgaged property and of mortgages, as commonly provided for in American States.
  2. It has been said that the taxation of merchants’ stock in trade in Massachusetts, by assessors’ estimate, if effect proceeds in a somewhat similar fashion to that of the French impôt des patentes and of the Prussian business tax. Why? or why not?
  3. Are there good reasons for taxing funded incomes at a higher rate than unfunded?
  4. It has recently been proposed in Great Britain to impose a general tax on property, based on the income tax returns, and levied at the rate of (say) five per cent. on the income derived from the property; reducing at the same time the income tax to one-half its present rate. Point out what important changes in the British tax system would result; consider what examples in other countries may have suggested the proposal: and give an opinion as to its expediency.
  5. What do you conceive to be the “compensatory theory” in regard to progressive taxation?
  6. What reasoning pertinent in regard to the principle of progression in taxation is also pertinent in regard to taxes on successions? in regard to the single tax?
  7. As between owner and occupier of real estate who is responsible for local rates in England? for local taxes in the United States? Do you believe that the differences have important consequences in the incidence of these taxes?
  8. Consider points of resemblance, points of difference, in the modes in which the States of Massachusetts and Pennsylvania tax (1) domestic corporations (2) the securities issued by foreign corporations.
  9. What grounds are there in favor, what against, the imposition of income taxes by the several States?

Source: Harvard University Archives. Harvard University, Mid-year examinations, 1852-1943. Box 4. Bound Volume: Examination Papers, Mid-Years 1897-98.

Also: Harvard University Archives. Examination Papers, 1873-1915. Box 5, Examination Papers 1898-99, Bound Volume, pp. 47-48.

Also: Harvard University Archives. Examination papers in economics 1882-1935, Prof. F. W. Taussig. Scrapbook. (HUC 7882), p. 61.

_______________________

Financial Administration and Public Debts
Economics 72

*72 hf. Financial Administration and Public Debts. Half-course (second half-year). Tu., Th., Sat., at 11. Professor [Charles Franklin] Dunbar.

Course 72 is devoted to an examination of the budget systems of leading countries, and their methods of controlling expenditure, the methods of borrowing and of extinguishing debts practised by modern states, the form and obligation of the securities issued, and the general management of public credit.

Topics will be assigned for investigation by the students, and a list of topics, references, and required reading will be used.

Source: Harvard University. Faculty of Arts and Sciences. Division of History and Political Science Comprising the Departments of History and Government and Economics, 1897-98, p. 38.

Enrollment data not published for 1897-98.

1897-98.
Economics 72
[Final Examination]

  1. What are the comparative advantages of (a) an Independent Treasury like that of the United States, and (b) the use of a bank or banks by the government, as practised in England or Germany?
  2. What changes (if any) of constitution, law or practice would be required, in order to establish a thorough-going budget system in the United States?
  3. Compare the French budget procedure with the English, and point out their respective advantages or disadvantages.
  4. Suppose a fiscal year to have ended before financial measures for the new year have been agreed upon. How would current expenditure be provided for in the United States? In England? In France? In Germany?
  5. What is the practice of those four countries respectively as regards the control of revenue by means of annual grants?
  6. Suppose the case of a country having a depreciated paper currency, but expecting the ultimate resumption of specie payments, and compelled to borrow on a large scale. Which method of borrowing upon bonds (principal and interest payable in gold) would be the best,—
    (a), To sell the bonds for par in gold and make the rate of interest high enough to attract buyers;
    (b) To sell the bonds for gold at such discount as might be necessary, their interest being fixed, say, at six per cent;
    (c) To sell the bonds for their nominal par in depreciated paper. Give the reasons for and against each method.
  7. State the probable effect on the selling value of bonds when their terms provide for, —
    (a) Annual drawings by lot for payment;
    (b) Reserved right to pay at pleasure after some fixed date;
    (c) Obligation to pay at some fixed date;
    (d) “Limited option” like that of the “five-twenties.”
  8. Examine the reasoning involved in the following expression of opinion:—
    “There is one essential difference between the anticipation of interest. payments, and the anticipation of the payment of the principal of a debt by purchases on the market. This latter procedure…requires a larger sum of money to extinguish a given debt than will be required after the debt comes to be redeemable; but no such result follows the anticipation of interest-payments. These are determined by the terms of the contract, and may be calculated with accuracy. The interest does not, like the market value of a debt, fall as the bonds approach the period of their redemption, and it is but the application of sound business rules to use any surplus money on hand in making advanced payments of interest.”
  9. Describe the existing arrangements for the reduction of the English debt.
  10. State, with reasons, your own conclusion as to the real advantage (if any) derived from the system of terminable annuities.

Source: Harvard University Archives. Examination Papers, 1873-1915. Box 5, Examination Papers 1898-99, Bound Volume, pp. 48-49.

_______________________

Banking and the History of the leading Banking Systems
Economics 122

*121 hf. Banking and the History of the leading Banking Systems. Half -course (first half-year). Tu., Th., Sat., at 11. Professor [Charles Franklin] Dunbar.]

[Note: originally announced as omitted for 1897-98.]

In Course 12[1] the modern system of banking by deposit and discount is examined, and its development in various countries is studied. The different systems of note-issue are then reviewed and compared, and the relations of banks to financial crises carefully analyzed. Practical banking does not come within the scope of this course. The study is historical and comparative in its methods, requiring some examination of important legislation in different countries, practice in the interpretation of banking movements, and investigation of the general effects of banking. The course, therefore, naturally leads to an examination of the questions now raised as to bank issues in the United States.

Source: Harvard University. Faculty of Arts and Sciences. Division of History and Political Science Comprising the Departments of History and Government and Economics, 1897-98, pp. 38-39.

Economics 12: Enrollment

[Economics ] 121. Professor Dunbar.—Banking and the History of the leading Banking Systems. Hf. 3 hours. 1st half year.

Total 12: 5 Graduates, 4 Seniors, 2 Juniors, 1 Other.

Source: Annual Reports of the President and the Treasurer of Harvard College 1897-98, (Cambridge, Massachusetts, 1899), p. 78.

1897-98.
Economics 121.
[Mid-year Examination]

A.

Give ONE THIRD of your time to these two questions.

  1. Suppose that, in the period 1848-70, India had had a banking system as extensively used and as efficient as that of England or the United States, and that in the East prices had depended upon competition as much as they did in the Western nations? How would these altered conditions have affected the drain of silver to India, and the value of the precious metals in America and Europe?
  2. What do you say to the general proposition, that England, “being a debtor nation,” can draw gold at pleasure from any part of the world?

B.

  1. A few years ago an American writer said:—
    “We will be able to resume specie payments when we cease to rank among the debtor nations, when our national debt is owed to our own people, and when our industry is adequate to the supply of the nation’s need of manufactured goods.”
    To what extent should you regard the circumstances of the resumption in 1879 as a verification of the reasoning implied in this statement?
  2. In what way did the payment of the French Indemnity, 1871-73, tend to stimulate affairs in England, Austria, and the United States?
  3. What economic conditions or events tended to make the year 1890 a turning point, both in domestic and in international finance? Give a clear statement of such as you recall.
  4. How do the banking and currency systems of England, France and the United States differ, as regards their ability respectively to resist export movements of gold?
  5. What temporary changes in the general level of prices in this country should you expect to see, as the result of a large permanent withdrawal of foreign capital? What ultimate change of prices should you expect?
  6. State the general conditions which determine the movement of gold as it issues from the mining countries and is distributes over the world?
  7. Cairnes discusses some of the conditions which determine the relative quickness with which countries raise their general scale of prices when a rapid depreciation of gold is in progress. Consider how far the effect upon a given country would be influenced by the fact that its exports were

(a) chiefly manufactured articles;
(b) chiefly articles of food.

Source: Harvard University Archives. Harvard University, Mid-year examinations, 1852-1943. Box 4. Bound Volume: Examination Papers, Mid-Years 1897-98.
Also: Harvard University Archives. Examination Papers, 1873-1915. Box 5, Examination Papers 1898-99, Bound Volume, pp. 51-52.

_______________________

International Payments and the Flow of the Precious Metals
Economics 121

[Was not offered first nor second term, instead see above]

[* 121 hf. International Payments and the Flow of the Precious Metals. Half-course (first half-year). Tu., Th., Sat., at 11. Professor [Charles Franklin] Dunbar and Mr. [Hugo Richard] Meyer.

Course 121 is taken up with the discussion of the movements of goods, securities, and money, in the exchanges between nations and in the settlement of international demands. After a preliminary study of the general doctrine of international trade and of the use and significance of bills of exchange, it is proposed to make a close examination of some cases of payments on a great scale, and to trace the adjustments of imports and exports under temporary or abnormal financial conditions. Such examples as the payment of the indemnity by France to Germany after the war of 1870-71, the distribution of gold by the mining countries, and the movements of the foreign trade of the United States since 1879, will be investigated and used for the illustration of the general principles regulating exchanges and the distribution of money between nations.

Source: Harvard University. Faculty of Arts and Sciences. Division of History and Political Science Comprising the Departments of History and Government and Economics, 1897-98, pp. 38-39.]

_______________________

Selected Topics in the Financial Legislation of the United States
Economics 162

*162 hf. Selected Topics in the Financial Legislation of the United States. Half-course (second half-year). Tu., Th., at 2.30. Professor [Charles Franklin] Dunbar.

The topics for study in this course for 1897-98 will be: (1) The Legal Tender Issues of the Civil War; (2) Development of the National Banking System. Subjects will be assigned and reports called for, requiring thorough investigation in the debates of Congress and other contemporary sources of information, for the purpose of tracing the history and significance of the legislative acts to be discussed, and a close study of such financial and commercial statistics as may throw light upon the operation of the acts.

Arrangements will be made by which graduate students and candidates for Final Honors in Political Science may take this course in connection with the Seminary in Economics as a full course running through the year.

Source: Harvard University. Faculty of Arts and Sciences. Division of History and Political Science Comprising the Departments of History and Government and Economics, 1897-98, p. 39.

Economics 162: Enrollment

[Economics ] 162. Professor Dunbar.—Selected Topics in the Financial History of the United States. Hf. 2 hours. 1st half year.

Total 8: 3 Graduates, 3 Seniors, 2 Juniors.

Source: Annual Reports of the President and the Treasurer of Harvard College 1897-98, (Cambridge, Massachusetts, 1899), p. 78.

1897-98.
Economics 162
[Year-end Examination]

A.

Give one-half of the time allowed for this examination to the discussion of any two of the questions stated under B.

B.

Answer, with such fulness as the remaining time allows, those of the following questions which you have not selected for discussion under A.

  1. Rhodes (History of the United States since 1850, iii., 567) states as “the conclusion which it seems to me a careful consideration of all the facts must bring us to,” that “The Legal Tender act was neither necessary nor economical.”
    Discuss this conclusion.
  2. In December, 1868, Senator Morton introduced a bill providing that specie payments should be resumed, by the government July 1, 1871, and by the banks January 1, 1872, greenbacks ceasing to be a legal tender at the latter date; gold to be provided in the Treasury by the accumulation of surpluses and by the sale of bonds, but no greenbacks to be redeemed until the date fixed for resumption by the United States.
    What would have been the probable operation of such a measure?
  3. Sherman said in January, 1874,—
    “The plan, which in my judgment presents the easiest and best mode of attaining specie payments, is to choose some bond of the United States which in ordinary times, by current quotations, is known to be worth par in gold in the money markets of the world, where specie is alone the standard of value, and authorize the conversion of notes into it.”
    Discuss the probable working of such a plan, having in view also Mr. Sherman’s strong objection to a contraction of the currency
  4. Suppose an Issue department of the Treasury, completely separated from all other business, provided with an ample reserve and strictly limited to the exchange of coin for notes and notes for coin as required by the public; what would you say would then be the nature and the force of the objections, if any, to the permanent maintenance of our legal tender issues?
  5. The greenbacks having been regarded originally as the temporary element in our paper currency and the bank notes as the permanent element, what were the one or two great turning points in the development which reversed this relation?
  6. If the issue of bank-notes were made equally available for all parts of the country, so far as the requirements of the system are concerned, would the South and South West find themselves more amply provided with paper currency than at present?
  7. What in your judgment is the most important function discharged by banks in this country, and what is your estimate of the importance and practicability of national supervision of their discharge of that function?
  8. The act just passed by Congress to provide ways and means for the expenditures occasioned by the war, contains the following section:—
    “That the Secretary of the Treasury is hereby authorized and directed to coin into standard silver dollars as rapidly as the public interests may require, to an amount, however, of not less than one and one-half millions of dollars in each month, all of the silver bullion now in the Treasury purchased in accordance with the provisions of the act approved July 14, 1890, entitled “An act directing the purchase of silver bullion and the issue of Treasury notes thereon, and for other purposes, and said dollars, when so coined, shall be used and applied in the manner and for the purposes named in said act.”
    State carefully the use and application of the dollars thus required by the act of 1890.

Harvard University Archives. Examination Papers, 1873-1915. Box 5, Examination Papers 1898-99, Bound Volume, pp. 554-55.

_______________________

Economics Seminary
Economics 20

20. Seminary in EconomicsMon., at 4.30. Professors [Charles Franklin] Dunbar, [Frank William] Taussig, and [William James] Ashley, and Asst. Professor Edward Cummings.

In the Seminary the instructors receive Graduate Students, and Seniors of high rank and adequate preparation, for training in investigation and discussion. No endeavor is made to limit the work of the Seminary to any one set of subjects. Subjects are assigned to students according to their needs and opportunities, and may be selected from any of the larger fields covered by the courses in which stated instruction is given. They may accordingly be in economic theory, in economic history, in applied economics, in sociology, or in statistics. It will usually be advisible for members of the Seminary to undertake their special investigation in a subject with whose general outlines they are already acquainted; but it may sometimes be advantageous to combine general work in one of the systematic courses with special investigation of a part of the field.

The general meetings of the Seminary are held on the first and third Mondays of each month. The members of the Seminary confer individually, at stated times arranged after consultation, with the instructors under whose special guidance they are conducting their researches.

At the regular meetings, the results of the investigations of members are presented and discussed. The instructors also at times present the results of their own work, and give accounts of the specialized literature of Economics. At intervals, other persons are invited to address the Seminary on subjects of theoretic or practical interest, giving opportunity for contact and discussion with the non-academic world. Among those who thus contributed to the Seminary in 1895-97 were President Francis A. Walker, Dr. Frederick H. Wines, Mr. S. N. D. North, Mr. A. T. Lyman, Mr. E. W. Hooper, and Mr. F. C. Lowell.

In 1896-97 the Seminary had fifteen members, of whom twelve were Graduate Students, two were Seniors in Harvard College, and one was a Law Student. Among the subjects under investigation in that year were: The Woollen Industry in England during the 17th and 18th centuries; Over-production and Over-accumulation in Economic Theory; The Taxation of Sugar in the United States and in Foreign Countries; The National Banking System with regard to its operation in the West and South; The Financial History of the Pennsylvania Railway; The Financial History of the Union Pacific Railway; The History of Immigration into the United States.

Source: Harvard University. Faculty of Arts and Sciences. Division of History and Political Science Comprising the Departments of History and Government and Economics, 1897-98, pp. 39-40.

Economics 20: Enrollment

[Economics ] 20. Professors Dunbar, Taussig and Ashley, and Asst. Professor Edward Cummings.—Investigation of topics assigned after consultation.

Total 12: 11 Graduates, 1Senior.

Source: Annual Reports of the President and the Treasurer of Harvard College 1897-98, (Cambridge, Massachusetts, 1899), p. 78.

Members of the Harvard Economics Seminary, 1897-1898

https://www.irwincollier.com/harvard-members-of-the-economics-seminary-1897-1898/

Image Source: Harvard Hall (1906). From the Center for the History of Medicine (Francis A. Countway Library of Medicine).

Categories
Economists Gender Radcliffe Undergraduate

Radcliffe. Paul Sweezy’s blue eyes and a summary of economics courses taken by the Class of 1942.

Paul Sweezy by many accounts was a Paul Newman of academic economics. This is implicitly confirmed in the following text, written by one of his fans for the Radcliffe Class of 1942 Yearbook summarizing Harvard economics courses offered to Radcliffe women in the early years of WWII.

_______________________

Economics. Ec. A—Or is the business cycle necessary? Wages, interest, profit, rent—where that last five dollars went. If value equals distribution, why do we pay so much tuition?—Money and Banking, or How Professor Harris converts the American business man to Keynes.—Corporations. Dull? How could it be, considering its Social Significance, and Dr. Sweezy’s blue eyes.—Economic Theory—watch ring-master Chamberlin corral the whole economic system into ceteris paribus.—Ec. 18. We have to strike a defense note in these parlous times.—Did we say strike? Ec. 81, Labor Problems, led this year by Messrs. Healy and Hogan.

Source (Text and Image): Radcliffe College Yearbook, Class of 1942, p. 43.

Categories
Chicago Funny Business

Chicago. The Journal of Progressive Hedonists Against Radical Thought (P.H.A.R.T.), Rodney Smith & Roger Vaughan, 1971

During the first year and a half of their graduate studies in economics at the University of Chicago in the 1970s, Rodney Smith and Roger Vaughan collaborated in the publication of the (Monty Phython inspired) satirical Journal of Progressive Hedonists Against Radical Thought (a.k.a., P.H.A.R.T.). Smith and Vaughan came to Chicago from UCLA and Oxford, respectively, and clearly shared a common sense of smell. Both later worked as economists at the RAND corporation. Roger Vaughan was responsible for pen-and-ink artwork which will be featured in a later post. He passed away in October 2021, but Rodney T. Smith is alive and well, President of the economics and strategic planning consulting firm Stratton Inc.

The second issue of the Journal of P.H.A.R.T. transcribed for this post was found in Milton Friedman’s papers at the Hoover Institution.

Request: if any visitors to Economics in the Rear-view Mirror have a surviving copy of other issues of the journal or of Roger Vaughan’s Chicago artwork, please let it be copied/transcribed and/or posted here for the benefit of future generations of economists yet untrained.

_____________________________

Vol. 1, No. 2   May, 1971

P.H.A.R.T. EDITORIAL

Mother Apathy, blindfold across both breasts, was aroused, albeit temporarily, when it was announced recently that the dynamic new publication ‘P.H.A.R.T.’ (whose first edition is already a collectors’ item) had engulfed the revered, but aging, ‘P.E.C. Notes’. From the autumn of 1971, a spokesman said, both journals would be published jointly under the P.H.A.R.T. banner. The entire publishing world, normally oblivious to news worth printing, was thrown into confusion. Exclaimed a Time-Life editor, “I’m padlocking my staff to their desks by their private parts!”

Wall Street reacted with typical concern. Both P.H.A.R.T. shares changed hands many times, but speculation was terminated when the purpose of the flimsy pieces of paper was misunderstood and they were flushed into the Hudson.

Smiled a P.H.A.R.T. staffer from his tax-deductible air mattress in sun drenched Lake Michigan, “Anyone who can print the kind of *%¢#$* that we do, is bound to succeed in a place like the Economics Department.

Comparison with Hugh Hefner’s Empire is obvious, a comparison which seems even closer in light of P.H.A.R.T.’s intention of running a pull-out center-fold of economics books without covers.

Certainly the new combined journal should prove to be a bright super-nova in the Gutenberg Galaxy.

P.H.A.R.T. INTERVIEWS

Chicago Charlie: In this issue we are honored to have a pleasantly in depth interview with an obscure, but nevertheless vital, member of the economics community, an economics groupie. We all know that surfers are followed by beach bunnies, musicians are plastered by groupies, but few of us are aware that economists have their own followers who admire them for their abstractions. Enough of such preliminaries. Let’s get it on. How long have you been an economics groupie?

Economics Groupie: I was a know-nothing college dropout until I, sort of, wandered into this economics class and saw this guy draw this groovy diagram. Minutes later I was the virtuous path of abstraction wherein what I adored was emphasized, and what I abhorred was forgotten. Like, nobody else would have me.

C.C.: Very interesting. Which abstraction really turns you on. I mean, what do you really hold near?

E.G.: What to, duckie? I cherish the nominal versus the real distinction; compensated versus uncompensated elasticities, and market versus non-market activities.

C.C.: A strange collection. What exactly do you mean by the latter?

E.G.: Man, like sometimes it’s a drag to hurl my bod on the market just to get what the traffic will bare. Non-market activity can be really heavy, like just getting an economics biggy to fondle my copy of Smith’s ‘Wealth of Nations’, I mean….

C.C.: Thank you. Have you ever had any bad experiences… that is, as an economics groupie?

E.G.: Yes; whenever I attend conventions, like five people on the same abstraction…

C.C.: Who are your favorite personalities?

E.G.: Friedman’s money, Lewis’ labor, Becker’s fertility, Harberger’s compensated triangles, Chez’s jiggles, Nerlove’s heteroscedasticity, Johnson’s distribution, Zecher’s….

C.C.: Really! Familiarity breeds contempt!

E.G.: No, baby, just frustration.

C.C.: Well, I must draw this interview to a close.

E.G.: Regretfully, we never started.

 

P.H.A.R.T.’s NEWS IN BRIEF

Boston: Following on the immense commercial success of rock musicals ‘Hair’ and ‘Tommy’, the songwriting team of Samuelson and Solow is rumored to be working on a rock opera based on the formers’ best selling ‘Foundations of Economic Analysis’. The star part of ‘Negative Semi-Definite Matrix’ is rumored to be played by Samuelson himself, but speculation is rampant concerning the famous nude scene in which cross-elasticities emerge, bare to the world, from the matrix determinant.

Chicago: The long held belief that all economists orbit around Chicago was cast into serious doubt, when, with the aid of powerful econometric telescopes, at least 200 economists were observed going ‘round in circles in the Boston area. “We are submitting the model builder to close questioning”, said a spokesman for the Chicago Inquisition. “I think we can show that these so-called observations were: (a) never made, and (b) not ‘proper’ economists anyway.”

New York: Although much doubt has been cast on the ‘as if’ approach to economic theology, Chicago feels it has, at last, come up with vindication for its views. After experimenting with several hundred dogs on the top of the brand new Trade Center, a Chicago scientist reported, “After we held each dog some five feet from the parapet, every single canine behaved as if they fully understood the Newtonian laws of gravity.” Surveying the pulp-covered street, he added, “That’s exactly the kind of empirical data we like to build on.”

Houston: Initial reports on analysis on moon rock samples brought back by Apollo 11, 12, and 14 confirm the fact that the moon is made of money, spokesman announced here today. “At first glance”, he read from a prepared statement, “they appear to be 19th century British gold sovereigns, with a picture of Milton Friedman and the inscription ‘Veni, Vidi, Vici.’”

London: The real purpose of the American policy of benign neglect towards international payments, masterminded by Harvard’s Gottfried Haberler became evident today when an English postman reported being molested by “A kind of satan-like figure, with an American accent, shouting something about my soul and throwing dollars everywhere”. Under separate questioning, the Federal Reserve admitted to attempting to buy the world and promised that next time, its agents would behave with more discretion and dignity.

Philadelphia: The following rumor, entirely unsubstantiated by P.H.A.R.T. foreign correspondent in Philadelphia, is circulating concerning famed British economist, Sir Roy Harrod. It appears that Sir Roy, unadjusted to the American matriarchal society was accustomed to addressing a sexually integrated class at the University of Pennsylvania, as “Gentlemen”. As attendance dwindled he was faced one day with an entirely female class. He gazed around for a few seconds, and then left muttering, “Since there is no one here, I shall not lecture”. Norman Mailer, eat your heart out.

 

P.H.A.R.T.’s LIVES OF THE GREAT ECONOMISTS

In a bold attempt to instill in economists a sense of pride in the historical development of their discipline, P.H.A.R.T, brings you, in each issue, a brief sketch of one of the giants upon whose shoulders we stand.

Crasso the Greek (? – 410 BC)

It is fitting that the first reference we have to this great economist occurs in the poetry of Obesia, the Fat-Woman of the caves, in those oft-quoted lines.

“The hills are alive with the sound of Crasso,
As he and his acolytes perform the dance of the drachma.”

Already the young Crasso, and his ‘Crassic’ followers were famous for running naked into holy shrines and leaving the walls covered in ‘graphiti’ (or graphs) illustrating some of the new concepts they had discovered. It is to this early period that the world owes the famous Crassic doctrines.

“When prices rise, things tend to become more expensive.”
“What lays ahead of us is in the future.”

Little else is heard of Crasso until he sprung to fame when called to the service of the King of Sicily to fight a serious outbreak of inflation. He successfully stemmed the fearsome tide of price rises by offering 21,000 warm chicken livers to the God, Hypa (a method, incidently, the Federal Reserve is currently considering). His triumph was short lived and as serious food poisoning decimated the population, popular feeling ran against him and he was forced to dust his naked body with flour and escape disguised as a statue.

He moved to Lydia where he perfected a technique of depicting, with amazing clarity, small cameo-like pictures of unequalled pornography on round pieces of metal. His art work became immensely popular, and although known deprecatingly by the Greeks as “Khoynos Pornos” (Foreign Filth) they circulated in Crasso’s native land and became popularly known as Khoyns.

Immensely wealthy, he returned to his native Corrinth idling his time gambling on the innumerable Greco-Persian wars, and was bankrupt when he offered 100 to 1 on Persia at Salamis.

He never again achieved his former glory, and in spite of nearly discovering the formula for the velocity of money, inventing, during hard winter, the concept of a wages freeze, and writing a prodigious number of strange tracts he moved slowly downhill. He gave one or two guest lectures, but was jeered from the podium when he spoke, with missionary zeal, of money floating down from the heavens. He is last recorded as the tragic model for Thucydides’ description of the effects of the plague during the Peloponnesian Wars.

P.H.A.R.T. MISCELLANY

As Others See Us

(From Berman’s ‘The Underground Guide to the College of Your Choice’.)

“The graduate school here (Chicago) is the old apprenticeship type of learning. The difference between the University of Chicago and other universities is like the difference between English Justice and French Justice — at other universities the professors consider you innocent of stupidity until you prove otherwise while here you are presumed guilty of stupidity until you prove yourself innocent.”

* * *

We hope, in each issue, to bring you some of the ‘bon mots’ that are the obvious concomitant of a concentration of high powered minds.

Stigler: “The government take-over of the railways is a vain and abortive attempt to make the post office look efficient.”

Johnson (H.G.): “When a professor leaves M.I.T. for Harvard, the average intelligence in both places rises.”

Samuelson: “If you take a 15 trillion year plan, then the theorem is correct.”

* * *

P.H.A.R.T. would take great privilege in awarding its most treasured prize, a corroded plastic waterbed, to all those sophisticated individuals who manage to reduce student-faculty visits to informal economics seminars. At the risk of belaboring the obvious, P.H.A.R.T, would like to announce, that:

(a) the esoteric economic pun is the lowest form of humor;

(b) there just may be someone in the room who is less than enthralled by the ability to play ‘Obscure-Journal-Article-Snap’ with all comers:

(c) the breath-taking account of how to take-apart a speaker, complete with a 20 minute digression of ‘statistical- discrepancies-I-have-seen-through’, should be saved for the autobiography.

It takes considerable ignorance to assume that everyone present is fascinated by economics 24 hours a day. Those who do have nothing else to talk about should try not to see it as a virtue. A more comprehensive ‘weltanschauung’ would make brighter living.

It was not clear, when the fire and smoke-laden Delphic rumblings had passed, whether or not the Nobel Laureate had actually said anything. After following the pillar of fire around the campus like the children of Israel, the Economics Department received a ‘lecture’, an experience not unlike speed reading a Dictionary of Quotations while being assaulted by a poorly-programmed 360. The quality of evening speakers this year has rarely risen above the esoteric. To quote Leijonhuvfud: “If this is how economics develops — where will it end?”

* * *

P.H.A.R.T. REVIEW OF BOOKS

In response to our previous reviews, readers submitted the following:

The Sensuous Criminal, by G.G. (Alcatraz, 1971) . Discusses in frank, no-nonsense language, the implications of over fifty Neuman-Morgenstern utility functions for criminal behavior. Explains why criminals with Moebius-strip utility functions usually get caught; why most extortionists have homogeneous-of-degree-less-than-one time preference functions; and many, many more. J. Edgar Hoover loved it — you will too. yours for only U= a + bI dollars!

The Godfather (A Story of Money, Interest, and Prices), by Donaldo Patinkini (Extortion Press, 1971). An inside man reveals the true interworkings of a powerful group of variables. Gives a detailed but chilling analysis of how inept government has allowed the Ma Fed to extract millions from society through monetary control.

The Holy Bible (The King Milton Version) Heavenly Press, 1971. In the beginning God created money. That took 6 days, on the 7th day he rested, and so nothing else matters.

 

P.H.A.R.T. CONTRIBUTIONS TO ECONOMIC THOUGHT

A Scatological Theory of Nominal Income*

Few things stand the economics profession in such shame than the narrowness of the treatment accorded to the determination of nominal income. The intensity of the Keynesian/Monetarist debate has obscured the more fundamental limitations of the logical possibilities examined.

It is the contention of this paper that this entire debate is hollow, and it is to the agricultural sector and the determinants of the demand for guano1 that we must look for any truly logical transmission mechanism. In Section 1 of this paper, below, the basic model for the determination of income is outlined, while Section 2 examines some of the implications for the American economy today.

Section 1: The Model

The rapid pace of industrialization in England has long proved a fertile hunting ground for the economist anxious to achieve tenure. Weber blamed it on the Church, Marx on the greedy bourgeois, McLuhan on the printing press, and Rostow argued that take-off was a result of take-off. Few have linked the unbinding of the industrial Prometheus to the discovery of guano deposits in Peru by strong-stomached British sailors in the first half of the 19th century.

Consider the fundamental identity,

Q= k Sh. Y

where Q is the quantity of agricultural output, Sh. is the quantity of soil nutrients, and Y is the quantity of land. It should be obvious that an increase in Sh., holding Y and k constant, will increase Q.2 It is not for nothing that the favorite Anglo-Saxon toast has come to be: “May the Bird of Paradise add nutrients to your soil.”

Neither is it a coincidence that British development should slow adown disastrously in the later 19thcentury when this supply dried up, and the wily Peruvians, under a seagull dictator, clamped a high tariff on this most valuable of bowel-movements.

Section 2: Post Guano Ergo Propter Guano?

It is only recently that the amazing correlation between the depth of guano deposits and the rate of growth of real output has received the attention it richly deserves. Since it avoids the quagmire of nominal and real debates, so long the bane of monetarism, the Federal Reserve has recently taken to publishing guano depth-counts taken at strategic points along the New England coast, and is currently contemplating setting up manure counting stations (M1 and M2) in. Nebraska and New Jersey. However, much pseudo-scientific pressure is levelled against this approach by economists who should know better, and there has been not a little reluctance to admit to using it. Few people, in fact, realize that the oft-repeated ‘1065’ figure was, in fact, the depth to which a now-famous aide to the budget supervisor sunk (in millimeters)3 when supervising one the first ‘counts’.

The theory leads to some important policy regulations:

(1) The removal of the welfare distorting Regulation P, by which a 10¢ surcharge is levelled on guano booths throughout the country.

(2) The conversion of banks into mere guano warehouses, to act as receptacles for such deposits and withdrawals, with a 100% reserve requirement.

(3) The automatic issuing of Feen-A-Mints with food stamps.

(4) The regulation of the guano growth rate to a fixed 5% per annum by an intensive, federally sponsored, seagull training program.

(5) The appointment of seagulls to replace Connally, Schultze, and Nixon to engender a harmonious, non-political environment in which to stabilize the economy.

It is hoped that this analysis proves a fertile hunting ground for the development of economic theory.

*The original idea was suggested by John Graffiti, although the author accepts all credit for the penetrating analysis. The future of 5 assistants is inextricably entwined with any potential faults.

1 Guano (Spanish). Literally, “the food from the heavens” or, more colloquially, “birdshit”, (Ed.).

2 For this proof, the author is indebted to a small, but persistent, Lake Michigan seagull.

3 Believed to proximate the distance from his feet to his neck, (Ed.).

 

P.H.A.R.T. PUZZLE CORNER

Answers to last issue’s questions:

  1. (a) False

(b) Uncertain. Such activity would involve income redistribution from bakers to hens.

(c) False. 1.5976 is, of course, the nominal number of students, which would be about 0.002 in real terms.

(d) True.

  1. (a) What do you receive when purchasing a buffalo?

(b) Which of the following can be categorized as an inferior good: sex or drink?

(c) What is the equilibrium weekly wage of an M.B.A.?

(d) Which?

  1. The quote, of course, was Keynes, page 40, and he was illustrating the importance of choosing units.
  2. H.A.R.T. FORKED TONGUE AWARD remains unawarded.
  3. H.A.R.T. MOST PERFECT COMPETITOR TROPHY is awarded to that perceptive reader who pointed out that the market for splinters from the cross of Christ would seem to fulfill Smith’s conditions an infinite number of buyers, a large number of suppliers, and an amazing invisible hand. More copies of this beautiful sculpture remain to be won for more insights into ‘Our Competitive Environment’.

This issue’s questions:

  1. Quote of the month. What amazing economist wrote the following and where?

“latex$ \left( X^{\prime }_{\ast }X_{\ast }\right)^{-1}  X_{\ast }\bar{y}&s=2$”

(This is to test that you are doing your reading.)

  1. TRUE, FALSE, IGNORANT.

Your grade will be largely independent of anything you write.

(a) Since when care packages were dropped into prisoner of war camps it was cigarettes and not prunes that were adopted as currency: prisoners were not acting rationally.

(b) Three helicopters in formation at sunset is an omen of inflation.

(c) Racehorses sometimes earn tens of thousands of dollars in stud services. Since economists are rarely paid as much for similar services, they are either being exploited, or earn non-pecuniary benefits.

  1. An economist of repute defined homotheticity in the following way: “You stand at the origin and jiggle your head this way and that way and nothing changes.” Utilizing this definition, determine whether the following is true. A compensated jiggle of the head will cause a non-homothetic function to appear homothetic.
  2. If Irving Fisher had defined the money stock as M7, then the business cycle would be the dance of the 7 veils?

* * *

SHELDON DE F’ART SAYS : “PER ARDUA AD INSOMNIA”

* * *

P.H.A.R.T, is an underground journal of negligible literary merit dedicated to the proposition that some followers of economics may possess a sense of humor. It is hoped to stimulate everyone into some form of response. For those who are leaving and wish to subscribe, the cost is $1.00 per annum, plus postage, for 8 copies (or more)

Most of the blame, any personal inquiries, submissions, letters, or donations should be directed to Rodney Smith or Roger Vaughan.

P.H.A.R.T.
Box P
Department of Economics
Social Science Building
West 59th Street
Chicago, Illinois 60637
U.S.A.

Source: Hoover Institution Archives. Papers of Milton Friedman, Box 79, Folder “79.6 University of Chicago Miscellaneous”.

Categories
Gender Race Radcliffe

Radcliffe, A.B. cum laude. African-American teacher of education. 1905

By chance I found the following verse given to a member of the Radcliffe class of 1905, Evangeline Rachael Hall (1882-1947), who was one of Radcliffe’s early African-American alumnae. While the verse testifies to her active participation in Radcliffe economics class(es?), it turns out that she went on instead to a long career at the oldest historically black college, Cheyney State Teachers College, where she taught education and English. Maybe a century later she would have continued studying economics.

In any event we all know how some of our economics students “only dully stare, or sit and sigh” and how few are “keen” and ask “The ‘how’ and ‘why things were’.”

___________________________

Radcliffe Yearbook 1905

Evangeline Rachael Hall, 39 Parker St., Cambridge

In economics she was keen and asked the “how”
and “why
Things were:” while some did only dully stare,
or sit and sigh.

Source: Radcliffe College. Book of the Class of Nineteen Five. (Boston: Everett Press, 1905), p. 27.

*  *  *  *  *  *  *  *  *  *  *

Evangeline graduated A.B. cum laude, June 27, 1905.

SourceAnnual Report of the President of Radcliffe College, 1904-1905, p. 24.

___________________________

Taught English at the Cheyney Training School for Teachers

Hall’s Special Work in English
(as of 1914/14)

(1) Public Speaking Summer Course at Emerson’s College of Oratory; (2) Composition and Literature, Harvard Summer School, 1912; (3) The Teaching of Oral English, Harvard Summer School, 1913.

Source: Annual Report of the Cheyney Training School for Teachers 1914/15, p. 8.

___________________________

Portrait from ca. 1930

Detail from an oil on canvas portrait painted by Laura Wheeler Waring in her style for portraits of leading African Americans.

Evangeline Rachel Hall, painted around 1930, portrays the educator and Wheeler’s long-time colleague at Cheyney as a mature, refined and educated woman, serene in her heritage and her achievements. Her pale, lightly patterned dress, draped on even paler skin, and her elongated face and pursed mouth give the impression of purity and untouchable respectability. She wears a corsage, not unlike an honored member of an exclusive sorority. She is clearly a member of the black middle class, the kind of person who inhabited Wheeler’s personal and professional sphere.”

SourceLife of a Portrait: Laura Wheeler Waring’s Anna Washington Derry by Valerie Harris in Pennsylvania Heritage, Summer 1919.

___________________________

A Pair of Obits

Miss Evangeline R. Hall

Funeral services for Miss Evangeline R. Hall, member of the faculty of Cheyney State Teachers College, Cheyney, Pa., will be held at 3 o’clock tomorrow afternoon at the Bell Funeral Home, 909 Poplar Street.

Miss Hall, a native of Cambridge, Mass., died Sunday in Delaware Hospital. She received her B.A. degree with honors from Radcliffe College, and did graduate work at Harvard University where she won her master of arts degree. She had taught at Cheney for 42 years.

Surviving are: A sister, Mrs. Madeline Hall Wheeler, this city; two nieces, Mrs. Mary Ann Avent, Buffalo. N. Y., and Mrs. Wheeler Murphy of Baltimore, and a nephew, Arthur E. Wheeler, Jr, this city.

Source: The Morning News (Wilmington, Delaware), 2 Dec 1947, p. 4.

*  *  *  *  *  *  *  *  *  *  *

Miss Evangeline Hall

Miss Evangeline Rachel Hall, connected for more than 42 years with the State Teachers College at Cheyney, Pa., died Sunday morning in Delaware Hospital, Wilmington, Del.

Miss Hall was director of the Coppin Laboratory Practice School and a teacher of education and English at the Cheyney School. She held an A.B. degree from Radcliffe College and a Master of Education degree from Harvard University.

Funeral services will be held at 3 P. M. today at 909 Poplar st., Wilmington. A memorial service will be held in Pennsylvania Hall at the college at 3:30 P.M. Sunday.

Source: The Philadelphia Inquirer, 03 Dec 1947, p. 32.

Image Source: Radcliffe College. Book of the Class of Nineteen Five. (Boston: Everett Press, 1905).

Categories
Economist Market Economists Johns Hopkins Yale

Yale. Evsey Domar’s Letter of Support for Promotion of Thomas Schelling to Full Professorship, 1957

For anyone whose experience in academic hiring and promotions has only been acquired over the past several decades, it might come as a shock that outside letters to support a department’s vote to offer a full professorship back in the 1950s would hardly exceed the length of a very modest thread of tweets today. To be honest, a thumbs-up emoji would have been an adequate response to Yale’s request for Evsey Domar’s opinion on the work of Thomas C. Schelling. 

Since the two letters transcribed for this post are so short, I figure that this is as good an opportunity as any to add a brief bio written for the 1962 Radcliffe Yearbook. The poor quality of the yearbook image is a pity, but at least we have a classic Harvard professorial pose complete with a bow-tie and a cigarette held à la Madmen.

_____________________________

From the 1962 Radcliffe Yearbook

THOMAS C. SCHELLING, Professor of Economics, graduated from high school just after the Great Depression. Upon entering the University of California in Berkeley, he decided to major in economics: “Somehow I felt that the social conflicts, the severe poverty, even the problems of war, were partly solvable by a knowledge of economics.” He graduated with an A.B. in 1944 and got his Ph.D. at Harvard in 1951.

Professor Schelling’s varied career background includes two years with the Marshall Plan (in Copenhagen and in Paris, 1948-50); Associate Economic Adviser to the Special Assistant to the President (1950-51); Officer-in-charge, European Program Affairs, Office of the Director for Mutual Security, Executive Office of the President (1951-53); Yale University (1953-58); the RAND Corporation (1958-59). He has been at Harvard since 1959, on the faculty and says, “Harvard students are more interesting to teach than those at Yale.”

Primarily interested in the relationship between economics and national security, Professor Shelling recently collaborated on Strategy and Arms Control, published in 1961. Other works include National Economic Behavior, International Economics, and numerous articles in various periodicals.

Although teaching and consultation in foreign policy (he is a member of the Scientific Advisory Board, U.S. Air Force) take up most of his time, Professor Shelling is now turning his research interests to the problems of bargaining and conflict management, particularly as these problems affect foreign affairs.) Professor Schelling feels that, although a nuclear test moratorium would be a good thing, test bans without some system of control or inspection are unworkable. Furthermore, he feels that cessation of tests alone is not a potent form of disarmament. As for the testing itself, we don’t really know whether testing is necessarily harmful.

Source: The 1962 Radcliffe Yearbook, p. 91.

_____________________________

Yale Requests Domar’s Opinion of Schelling

Yale University
Department of Economics
New Haven, Connecticut

Lloyd G. Reynolds, Chairman

February 18, 1957

Professor Evsey Domar
Department of Political Economy
Johns Hopkins University
Baltimore 15, Maryland

Dear Evsey:

The Department here has voted to promote Thomas C. Schelling to the rank of Professor of Economics. We are now about to begin putting the appointment through the regular committee procedures. It is customary at this stage to invite a number of leading scholars in other institutions to appraise the qualifications of the candidate. I should be grateful if you could take time to write me your impression of Schelling—the quality of his thinking and scholarship, his probably contribution to economics over the long run, his professional standing in comparison with other men of about his own age, and his general suitability for a professorship here.

We shall value your judgment and I am sure will find it helpful in putting the matter before our faculty for action.

Sincerely yours,
[signed] Lloyd

LGR/shd

_____________________________

Copy of Domar’s Response

25 February 1957

Professor Lloyd G. Reynolds
Chairman
Department of Economics
Yale University
New Haven, Connecticut

Dear Lloyd:

This is in response to your letter of February 18 regarding the qualifications of Thomas C. Schelling.

I have known him approximately since 1944 or 1945 and have read most of his writings. He is an exceptionally capable young man, endowed with creative intelligence and with common sense. I have the highest opinion of him as an economist and great hopes regarding his contribution to economics.

In comparison with other men of his age he stands out very close to the top. I would support his promotion most wholeheartedly.

Sincerely yours,

Evsey D. Domar
Professor of Political Economy
The Johns Hopkins University
(on leave, spring term, 1956-57)

EDD:am

Source: Duke University. David M. Rubenstein Rare Book & Manuscript Library, Economists’ Papers Archive. Evsey D. Domar Papers, Box 8, Folder “Yale University (1 of 2)”.

Image Source: Thomas Schelling portrait, 1964. Harvard University. Office of News and Public Affairs. Hollis Images olvwork369281.

Categories
Economics Programs Economists Harvard Teaching Undergraduate

Harvard. Promotion for Harold H. Burbank, Job Offer for Allyn Young 1919

This provides some back-story to the rise of Harold Hitchings Burbank in the Harvard economics department. Coincidentally, some light is cast on the salary negotiations involved in the hire of Allyn Young, as well as the hopes the department of economics held in the prospect of Young joining the economics department.

Chairman Bullock’s characterization of Burbank “He does everything willingly, but we are already in danger of driving the willing horse to death” is not exactly the language a chairman today would use today to justify a promotion for an assistant professorship…I hope.

___________________________

Harvard University
Department of Economics

F.W. Taussig
T.N. Carver
W.Z. Ripley
C.J. Bullock
E.F. Gay
W.M. Cole
O.M.W. Sprague
E.E. Day
B.M. Anderson, Jr.
J.S. Davis
H.H. Burbank
E.E. Lincoln

Cambridge, Massachusetts
12 o’clock. January 28, 1919.

Dear Mr. Lovell:

I have failed thus far to get in touch with Dr. Burbank, but will leave word at his house, and he will doubtless come to see you tomorrow.

I wish to express the hope that you will not propose any arrangement to him by which he will have to do any more work or make any more labor-consuming adjustment in connection with his work this year. He does everything willingly, but we are already in danger of driving the willing horse to death. Your suggestion that recent graduates now studying in the Law School be put in to do section work in Economics A. involves, even tho these new men are placed in charge of sections which began work in September, an amount of labor, responsibility, and worry on Burbank’s part which I feel strongly It would be unfair to ask of him.

I have not myself been one of the real sufferers from the war, so far as University work is concerned. Such extra work as I have had to do for the men in Washington has been comparatively limited in amount, and some of my ordinary work has been decreased so that I have not suffered greatly. But the younger men who have stood by us have had a bad time, and I feel so keenly that it is unjust not to give them relief as soon as we can do it that I hate to think of Burbank’s being asked to make any further readjustments in Economics A.

You will recall, if you will review the last two years, that I have not found difficulties in the way of doing the things which it was necessary to ask the Department to do, and have been ready to disorganize, or readjust and adapt, to any necessary extent. I have further found the ways of doing this; and only last fall, in spite of the fact that I felt it was hardly right for Day to be taken from us, I went to a deal of trouble to fix up an arrangement under which he might be released. If I saw any arrangement now, I would surely make it, as I have done in the past. If Burbank can think of any arrangement that I have not been able to think of, I shall be glad to have it put into effect; but I wish to represent to you that it will not only be bad for the course, but very unfair for Burbank to ask him to take young and inexperienced instructors whose heart is in the Law School work anyway, and fit them into section work in Economics A at this time. Moreover, this arrangement involves delay of at least ten days or a fortnight, and our men need relief at the earliest moment. There are certainly no suitable men in the Law School now; and if any register next week, it will take time to find them out, to make arrangement, and to have them get up their work so that they are fit to take charge of a section. should think that under this plan it would be more rather than less than a fortnight before our men would get any relief. If you could know from actual contact with conditions what I have been compelled to know about the work of our young men during the war, I believe you would feel as strongly as I do that what they need now is immediate relief and not a plan by which they will have to spend the next month breaking in green, and possibly inefficient, substitutes. By the time that Burbank gets Economics A running smoothly again, if, indeed, that can be done at all, the term will be most over and the acute need of relief will be almost at an end.

Sincerely yours,
[Signed] Charles J. Bullock

President A. Lawrence Lowell

___________________________

Harvard University
Department of Economics

F.W. Taussig
T.N. Carver
C.J. Bullock
E.F. Gay
W.M. Cole
O.M.W. Sprague
E.E. Day
J.S. Davis
H.H. Burbank
E.E. Lincoln

Cambridge, Massachusetts
March 8, 1919.

My dear Mr. Lovell:

Dr Burbank informs me that he has received from Dartmouth College the offer of a full professorship, and this makes it necessary for the Corporation to consider whether it desires to retain him at Harvard. You will recall that two years ago the Department of Economics recommended that Burbank be advanced to an assistant professorship. This was at the time when he received from Chicago University the offer of an assistant professorship with full charge of their instruction in Public Finance. A year ago I brought the matter to your attention, but you desired to postpone action until Burbank’s book had been published. Last June I asked whether you would be willing to waive the question of publication of Burbank’s book, which was nearly, but not quite, completed. in order that he might accept employment from a committee of the American Economic Association, which would both be remunerative and give him an unusual opportunity to investigate a subject in which he is greatly interested, namely, the practical operation of the Federal income and excess profits taxes. You sent me word through Mr. Pierce that you would waive the requirement, and that you would be glad to have Mr. Burbank accept this employment.

Mr. Burbank made a distinct success of his work for the Economic Association, and such success as the Committee achieved was largely due to him. This year he has been conducting Economics A, and has demonstrated his ability to handle that course in a satisfactory manner. It seems to me that he is an invaluable man for the Department, and I hope that the Corporation will be able nor to advance him to an assistant professorship.

You also asked me this morning to write you concerning Allyn A. Young, whom we have had under consideration for a number of years.

In the winter of 1916-17 the full professors of the Department of Economics, after carefully looking over the field, recommended to you that Mr. Young be called to a full professorship at Harvard University.

You authorized me to write to Young and inquire whether he could be secured, and if so, at what salary; and I was able to report to you that Young would come to Harvard if he were offered a full professorship at a salary of $4500. At this juncture the United States entered the war, and the matter was necessarily dropped.

Last December Professors Gay and Haskins called my attention to the fact that Young was likely to receive an offer from Columbia University, and I held a hurried conference with them, and they later conferred with you. Action was postponed, inasmuch as Mr. Young was going to the Peace Conference as exert on economic resources; and it appeared probable that, if we could offer him a professorship at $5000, we could secure him for Harvard, even tho another offer developed elsewhere.

I hope that the Corporation will feel able to extend a call to Professor Young at this time. Since I talked with you this morning, I have met Professors Carver and Ripley, and they both concur in the recommendation which I make. Professor Gay gave you his opinion in December; and since that time I have heard from Taussig, who still is of the opinion that we ought to call Young.

I have no further knowledge as to the amount of salary that it would be necessary to offer. I assume that we should have to offer at least $4500, which was the figure that would have been necessary in 1916; and in view of Young’s increased experience and enhanced reputation, I should think that a salary of $5000 would be justified.

It is, I believe, important for the Department to secure Young at this time. We had in 1917 a Department of Economics which was recognized as one of the strongest in the country; but we needed Young at that time, and shall need him still more now in order to develop our work during the next decade. With him, I believe we should have a department that would be recognized as very clearly the strongest department in the country.

There is one further consideration to be taken into account in connection with extending a call to Young. If our economic research enterprise proves permanent, Young would be absolutely the best man in the country to coöperate with Professor Persons in carrying through the work we have undertaken. With Young and Persons in the economic research undertaking, we should have almost a monopoly of high class statistical brains. Young’s appointment was recommended by the Department in the winter of 1916-17, before the Committee on Economic Research was established, and without any reference to the development of that Committee’s work. The Department recommended him because they thought he was the one man whom the Department needed. The point I am now making is that Young is the one man whom our economic research undertaking needs, so that it seems upon every account desirable to add him to our staff next fall. Under the arrangement that I have in mind, if our economic research enterprise proves permanent, Professor Persons could give two-thirds of his time to the Committee on Economic Research and one-third to teaching, and Professor Young could give two-thirds of his time to teaching and one-third to the Committee on Economic Research. By this arrangement the Department of Economics would gain two teachers of the very highest reputation at an expense amounting only to the salary of one full professor, while the Committee on Economic Research would secure the services of the two minds in the country which are best adapted for the immediate work it has in hand.

Sincerely yours,
[signed] Charles J. Bullock

President A. Lawrence Lowell

___________________________

Carbon Copy of Letter from President Lowell to Professor Bullock

March 8, 1919

Dear Mr Bullock:

I understand that Mr Burbank is feeling uneasy about his promotion, and has been made valuable offers from elsewhere. Mr Pierce, at my request, wrote you last May that the completion of his book was not essential to his promotion to an assistant professorship. He is as near as possible the soul of the body of tutors; and I think it is important that we should make it clear that good work as a tutor will receive as much recognition as an equally good conduct of lecture courses. Would it not be well, therefore, if Mr Burbank were appointed an assistant professor now? There is a Corporation meeting on Monday, and I should be very glad if you could communicate with me before it takes place, if you come home in time.

Very truly yours,
[stamp] A. Lawrence Lowell

Professor Charles J. Bullock
6 Channing Street
Cambridge, Mass.

Source: Harvard University Archives. President Lowell’s Papers 1917-1919. Box 124. Folder 1689.

Categories
Economics Programs Harvard Undergraduate

Harvard. Economics Department Committee Assignments, 1972-73

 

“The Division of Administration is limited by the Extent of the Department” might have been a chapter title in a history of economics written by Adam Smith were he to have lived two centuries after the publication of the Wealth of Nations. I transcribed and now post the following list of administrative committees/tasks and the names of members of the Harvard economics department belonging to each during the 1972-73 academic year.  The list comes from the economic historian Alexander Gerschenkron’s papers at the Harvard Archives. I have paired this artifact with the printed list of economics faculty teaching economics courses in 1972-73 from the annual Harvard course catalogue.

_____________________________

Committee Assignments for 1972-73

  1. Undergraduate Instruction

Otto Eckstein, Chairman
Elisabeth Allison
Abram Bergson
James Duesenberry
Jerry Green
William Raduchel
Martin Spechler
Francois Wilkinson

  1. Examing Committee

Benjamin Friedman, Chairman
Richard Caves
Abram Bergson
Robert Dorfman
Jerry Greene
Martin Spechler

Graduate Instruction

Robert Dorfman, Chairman
Truman Bewley
Hendrik Houthakker
Gregory Ingram
Richard Musgrave
Thomas Schelling
Gail Pierson

Mathematics Examination

Truman Bewley
David Starrett

Theory Examination

Arthur Smithies, Chairman
Zvi Griliches
Glenn Jenkins
John Lintner
Stephen Marglin
Janet Yellen
Tsuneo Ishikawa (Spring)

Quantitative Methods Examination

Lance Taylor, Chairman
Edward Leamer
William Raduchel
Howard Raiffa
T. N. Tideman
Thomas Horst (Spring)

Economic History Examination

Alexander Gerschenkron
Paul David

Ph.D. in Business Economics

John Lintner

Fellowships and Admissions

Zvi Griliches, Chairman
Elisabeth Allison
Hendrik Houthakker
T. N. Tidema
Marcelo Selowsky

Less Developed Countries Recruitment

Richard Mallon

Recruitment of Black Students

T. N. Tideman

  1. Non-Tenure Personnel

John Kain, Chairman
Richard Caves
James Duesenberry
Harvey Leibenstein
Marc Roberts
David Starrett
Janet Yellen

Placement Officer

Dwight Perkins

  1. Publications

Harvard Economic Studies

Richard Caves, Chairman
Edward Leamer

Quarterly Journal of Economics

Richard Musgrave, Editor

Review of Economics & Statistics

Hendrik Houthakker, Editor

  1. Wells Prize

Harvey Leibenstein, Chair
Alexander Geschenkron
Jerry Green

  1. Political Economy Lectures

Wassily Leontief, Chairman
Stephen Marglin

  1. Schumpeter Prize

Arthur Smithies, Chairman
Wassily Leontief

  1. Goldsmith Prize

Gregory Ingram, Chairman
Richard Musgrave

  1. Department Minutes

Martin Spechler

  1. Harvard Computing Center

William Raduchel

  1. Universities-National Bureau Committee

John Lintner

  1. Administrative Committee, Harvard Institute for Economic Research

William Raduchel, Acting Chairman
James Duesenberry
John T. Dunlop
Zvi Griliches
Dale Jorgenson (fall)
Dwight Perkins

Source: Harvard University Archives. Papers of Alexander Gerschenkron, Box 3, Folder “Economics (General) 1973/74, 1 of 2”.

_____________________________

1972-73
Faculty of the Department of Economics

James S. Duesenberry, William Joseph Maier Professor of Money and Banking (Chairman)

Elisabeth S. Allison, Assistant Professor of Economics

Kenneth J. Arrow, Professor of Economics (on leave 1972-73)

Abram Bergson, George F. Baker Professor of Economics

Truman F. Bewley, Assistant Professor of Economics and of Mathematics

Samuel S. Bowles, Associate Professor of Economics (on leave 1972-73)

Richard E. Caves, Professor of Economics

Hollis B. Chenery, Lecturer on Economics

David C. Cole, Lecturer on Economics

Paul A. David, Visiting Professor of Economics (Stanford University)

Kenneth M. Deitch, Assistant Professor of Economics (on leave 1972-73)

Robert Dorfman, David A. Wells Professor of Economics

John T. Dunlop, Lamont University Professor, Dean of the Faculty of Arts and Sciences

Otto Eckstein, Professor of Economics

Martin S. Feldstein, Professor of Economics (on leave 1972-73)

Benjamin M. Friedman, Assistant Professor of Economics

John Kenneth Galbraith, Paul M. Warburg Professor of Economics (on leave spring term)

James D. Gavan, Lecturer on Population Sciences (Public Health) Lecturer on Economics (spring term only)

Alexander Gerschenkron, Walter S. Barker Professor of Economics

Richard T. Gill, Lecturer on Economics

Carl H. Gotsch, Lecturer on Economics

Jerry Green, Assistant Professor of Economics

Zvi Griliches, Professor of Economics

Albert O. Hirschman, Lucius N. Littauer Professor of Political Economy (on leave 1972-73)

Thomas Horst, Assistant Professor of Economics (on leave fall term)

Hendrik S. Houthakker, Professor of Economics

Gregory Ingram, Assistant Professor of Economics

Dale W. Jorgenson, Professor of Economics (on leave spring term)

John F. Kain, Professor of Economics

Leonard Kopelman, Lecturer on Economics

Edward Leamer, Assistant Professor of Economics

Harvey Leibenstein, Andelot Professor of Economics and Population

Wassily W. Leontief, Henry Lee Professor of Economics

John Lintner, George Gund Professor of Economics and Business Administration (Business)

Arthur MacEwan, Assistant Professor of Economics (on leave 1972-73)

George F. Mair, Visiting Professor of Population Economics (Smith College)

Richard D. Mallon, Lecturer on Economics

Stephen A. Marglin, Professor of Economics

Albert J. Meyer, Professor of Middle Eastern Studies, Lecturer on Economics

Richard A. Musgrave, Harold Hitchings Burbank Professor of Political Economy; Professor of Economics (Law)

Gustav Papanek, Lecturer on Economics (on leave 1972-73)

Dwight H. Perkins, Professor of Modern China Studies and of Economics

Gail Pierson, Assistant Professor of Economics

William J. Raduchel, Assistant Professor of Economics

Howard Raiffa, Frank Plumpton Ramsey Professor of Managerial Economics (Business)

Marc J. Roberts, Associate Professor of Economics

Sherwin Rosen, Visiting Professor of Economics (University of Rochester)

Henry Rosovsky, Frank W. Taussig Research Professor of Economics

Michael Rothschild, Assistant Professor of Economics (on leave 1972-73)

Thomas C. Schelling, Professor of Economics

Marcelo Selowsky, Assistant Professor of Economics

Arthur Smithies, Nathaniel Ropes Professor of Political Economy

Martin C. Spechler, Lecturer on Economics and Social Studies

David A. Starrett, Associate Professor of Economics

Joseph J. Stern, Lecturer on Economics

Carl M. Stevens, Visiting Professor of Economics (Reed College)

Lance Taylor, Assistant Professor of Economics

T. Nicolaus Tideman, Assistant Professor of Economics (on leave fall term)

Thomas A. Wilson, Visiting Professor of Canadian Studies (University of Toronto)

Janet Yellen, Assistant Professor of Economics

Other Faculty Offering Instruction in the Department of Economics

Francis M. Bator, Professor of Political Economy (Kennedy School) (Public Policy, West European Studies)

Ralph E. Berry, Jr., Associate Professor of Economics (Public Health)

William M. Capron, Lecturer on Political Economy (Kennedy School) (Political Economy and Government)

Peter B. Doeringer, Associate Professor of Political Economy (Kennedy School)

Rashi Fein, Professor of Economics of Medicine at the Center for Community Health and Medical Care (Medicine)

Sherwood Frey, Assistant Professor of Business Administration (Business)

Herbert M. Gintis, Lecturer on Education (Education)

Joseph J. Harrington, Associate Professor of Environmental Health Engineering (Public Health)

Henry D. Jacoby, Associate Professor of Political Economy (Kennedy School)

James R. Kurth, Associate Professor of Government (Government)

Walter J. McCann, Jr., Associate Professor of Education (Education)

Harold A. Thomas, Jr., Gordon McKay Professor of Civil and Sanitary Engineering (Engineering and Applied Physics)

Richard J. Zeckhauser, Professor of Political Economy (Kennedy School)

Source: Harvard University, Faculty of Arts and Sciences. Courses of Instruction for Harvard & Radcliffe, 1972-73, pp. 183-185.

Image Source: Littauer Center (July 1970). Harvard University Archives.

Categories
Economics Programs Harvard

Harvard. Visiting Committee Reports. 1895, 1901, 1903, 1906, 1908, 1914

Around the turn of the 20th century visiting committees to the Harvard economic department were supposed to submit reports to the Board of Overseers of Harvard College at least once every three years. When I first saw that at archive.org there were six items listed dealing with Harvard’s visiting committees I was excited, hoping to find a rich mine of material as extensive as Arthur F. Brimmer’s 1974 Report of Economics Department Visiting Committee. As you can see from below, quite the opposite is the case. The early Visiting Committees submitted the nano-reports transcribed and posted below.

Perhaps one indication of the purely formal nature of the reporting system is that the “department of political economy” was renamed the “department of economics” in 1892 but the visiting committee was still referred to the visiting committee “on political economy” as late as 1912-1913.

__________________________________

LII.

REPORT OF THE COMMITTEE TO VISIT THE DEPARTMENT OF POLITICAL ECONOMY.
May 22, 1895

To the Board of Overseers of Harvard College:

ln the department of Political Economy the Professors feel the pressure of increased numbers of students, the lack of satisfactory space for lecture rooms, and the lack of time for independent work and research.

The difficulty is most serious in Economics 1. The lecture room available is ill-fitted for speaking or hearing, and the great number of students make it impossible, with the present corps of instructors, to divide them into sections small enough for adequate teaching and discussion. The difficulty has been met, as far as practicable, by strenuous and continuous work on the part of the Professors and instructors. This department, like others, feels the want of more books for reference, and of more library space for books and reading.

ARTHUR T. LYMAN, Chairman

Source: Harvard University. Reports of the Visiting Committees of the Board of Overseers of Harvard College from February 6, 1890 to January 8, 1902. (Cambridge, Massachusetts, 1902), p. 299.

__________________________________

XCIX.

REPORT OF THE COMMITTEE ON POLITICAL ECONOMY.
June 5, 1901.

To the Board of Overseers of Harvard College:––

The Committee on Political Economy has had this year and last year long conferences with the Professors in the Department, and members of the Committee have also corresponded with the head of the Department.

A very large proportion of the students take Political Economy, and many take the more advanced courses. The courses seem to be satisfactorily conducted and the lecture system is supplemented by conferences and teaching in sections.

A course in Accounting has been taken by a considerable number of students, and next year an excellent programme is proposed for a course in business law. It is intended that such special courses should be confined to graduates or to seniors, and it seems best that they should not interfere with the general and broader courses of instruction in this or any other department.

The great loss to the University and to the Department of Political Economy from the death of Professor Charles F. Dunbar has been referred to by the President in his report.

For the Committee,

ARTHUR T. LYMAN,
Chairman

Source: Harvard University. Reports of the Visiting Committees of the Board of Overseers of Harvard College from February 6, 1890 to January 8, 1902. (Cambridge, Massachusetts, 1902), p. 621.

__________________________________

CXXXIV.

REPORT OF THE COMMITTEE ON POLITICAL ECONOMY.
September 30, 1903.

To the Board of Overseers of Harvard College:—

The Committee on Political Economy met the professors and instructors.

Professor Taussig, who has been absent for a long time on account of illness, will resume a part of his work in October.

There continues to be a very large attendance in the courses of Economics — 519 in Economics 1, which deals with the general outlines of the subject, and one hundred or more in four other courses on banking, economic history of the United States, problems of labor and industrial organization, and currency legislation with recent experience and theory. These large classes involve the usual difficulties in giving interesting and adequate instruction, but they are gratifying evidences of the interest in this important subject. They make necessary many instructors for proper division of large classes, and consequently large expense.

The list of courses in the Catalogue shows the wide range of the instruction.

The Quarterly Journal of Economics, published by the Department, is well worthy of your attention. In recent numbers Professor Taussig has given an interesting and instructive history of the iron industry in this country, and the sugar problem has been fully treated.

Many other articles on important practical and theoretical questions have appeared in this Journal, which, like all publications of this class, needs the increased subscriptions and financial support to which its merits fully entitle it.

For the Committee,

ARTHUR T. LYMAN,
Chairman

Source: Harvard University. Reports of the Visiting Committees of the Board of Overseers of Harvard College from January 8, 1902 to July 30, 1909. (Cambridge, Massachusetts, 1909), p. 741.

Visiting Committee Members (1903):

Arthur T. Lyman, Charles S. Fairchild, Horace E. Deming, John E. Thayer, John F. Moors.

Source: Harvard University. Reports of the Visiting Committees of the Board of Overseers of Harvard College from January 8, 1902 to July 30, 1909. (Cambridge, Massachusetts, 1909),  following p. 708.

__________________________________

CLXXVIII.

REPORT OF THE COMMITTEE ON POLITICAL ECONOMY.
May 9, 1906.

To the Board of Overseers of Harvard College:—

The Committee on Political Economy met the Professors in the Department at a long conference. They are able, accomplished, and interesting, and there is, as usual, a very large attendance on their various courses of instruction. These courses deal not only with economic theory, but with transportation, finance, taxation, currency legislation, and banking, economic history, the distribution of wealth, labor, socialism, commercial law, and accounting.

The course on the principles of accounting is taken chiefly by Graduates and Seniors, and it is the purpose of the Department not to sacrifice to it the general and fundamental courses, but to confine it mainly to Seniors or Graduates rather than to follow the plan of some of the Western State Universities in which courses are given throughout the college terms treating in an elaborate way the practical details of business in various branches.

The large attendance in several of the courses in Economics requires subdivisions and conferences, and instructors for such divisions. There is naturally considerable difficulty in securing the desired or desirable instructors, owing to the small salaries that can be afforded and the consequent lack of permanency of employment, and it is much desired that the needed competency and permanency should, if possible, be secured by the assignment of some fellowships to the best of the instructors in the subdivisions and conferences. Much of the time of the Professors might be saved by the addition of various apparatus needed. Some help in this matter has been given by members of the Committee and others.

For the Committee,

ARTHUR T. LYMAN,
Chairman

Source: Harvard University. Reports of the Visiting Committees of the Board of Overseers of Harvard College from January 8, 1902 to July 30, 1909. (Cambridge, Massachusetts, 1909), p. 925.

Visiting Committee Members (1906):

Arthur T. Lyman, Charles S. Fairchild, Horace E. Deming, John F. Moors, J. Wells Farley.

Source: Harvard University. Reports of the Visiting Committees of the Board of Overseers of Harvard College from January 8, 1902 to July 30, 1909. (Cambridge, Massachusetts, 1909), after p. 843.

__________________________________

CCVIII.

REPORT OF THE COMMITTEE ON POLITICAL ECONOMY.
April 8, 1908.

To the Board of Overseers of Harvard College:—

The Committee on Political Economy met the Professors of the Department, and considers that the conditions are satisfactory.

There seems to be nothing new to report, except that arrangements have become possible through the generosity of various persons for carrying on for five years advanced business courses, such as have been under consideration for several years past.

More space is needed for conferences and books, and some plans are under consideration for relief in these matters.

For the Committee,

ARTHUR T. LYMAN,
Chairman

Source: Harvard University. Reports of the Visiting Committees of the Board of Overseers of Harvard College from January 8, 1902 to July 30, 1909. (Cambridge, Massachusetts, 1909), p. 1128.

Visiting Committee Members (1908):

Arthur T. Lyman, Frederic A. Delano, John F. Moors, Charles S. Fairchild, William Endicott, Jr., J. Wells Farley, Charles G. Washburn.

Source: Harvard University. Reports of the Visiting Committees of the Board of Overseers of Harvard College from January 8, 1902 to July 30, 1909. (Cambridge, Massachusetts, 1909), after p. 1118.

__________________________________

EXECUTIVE COMMITTEE.
REPORT OF THE SECRETARY FOR THE ACADEMIC YEAR 1913-14.
April 13, 1914.

5. Economics. — The enrolment of students in Economics for the first, half of the academic year shows a falling off from 1634 to 1520 students, the most significant loss taking place in the course on Banking. The amount of instruction offered at Harvard in Economics exceeds that offered at any other University except Michigan, while in the amount of money expended Harvard is far in the lead. The Committee notes the continuing eminence and prosperity of the Quarterly Journal of Economics, published by the Department since 1886, which maintains its place as one of the foremost periodicals in the subject in the English Language, and perhaps the foremost. It has the largest circulation of any periodical published by the University, except the Law Review, and there is no more certain way of making a reputation as a writer on Economic subjects than to publish an article in this journal. The most important need of the Department is a fund for the endowment of Economic Research, which will make possible the scientific investigation of pressing economic questions. A very interesting supplemental report is made by Mr. John Wells Morss, one of the members of the Committee, representing the views and experience of a man of business, rather than of a student of education. It should be read as a whole, as it is difficult to give a satisfactory impression of its interest and value by quotations or a summary.

Source: Appended to: Harvard University. Reports of the Visiting Committees of the Board of Overseers of Harvard College for the Academic Year, 1914-15. (Cambridge, Massachusetts, 1915), three pages after page 260.

Image Source: John Harvard statue, ca. 1904. U.S. Library of Congress Prints and Photographs Division.